CARDIAC PT 1

Réussis tes devoirs et examens dès maintenant avec Quizwiz!

xenograft

Graft from one species to another species (animal to man).

Direct Vasodialators

Hydralazine

lymphangitis

Inflammation of lymphatic channels or vessels.

Verapamil (Calan)

- Calcium channel blocker - Used to Tx angina, hypertension & arrhythmias

Angiotensin Receptor Blocker (ARB)

-sartan

Each small box on EKG paper represents

0.04 seconds

normal PRI (PR interval)

0.12-0.20 seconds

Each large square on EKG paper represents?

0.20 seconds

The nurse is evaluating a patient's preoperative teaching for a commissurotomy. The patient shows understanding of the purpose of this procedure by stating which of the following? 1. "Fused valve flaps are separated to enlarge the valve opening." 2. "A mechanical valve is inserted to replace a valve." 3. "The valve flaps are repaired or reconstructed." 4. "A biological valve is inserted to replace a valve."

1. "Fused valve flaps are separated to enlarge the valve opening."

The nurse is caring for a patient who has peripheral arterial disease. Which of the following statements by the patient indicates understanding of how to manage the pain of peripheral arterial disease? 1. "I will sit with my legs down." 2. "I will use a reclining chair." 3. "I will lie down frequently." 4. "I will do knee flexion exercises."

1. "I will sit with my legs down."

The intensive care department nurse is assessing the client who is 12 hours post-myocardial infarction. The nurse assesses an S3 heart sound. Which intervention should the nurse implement? 1. Notify the health-care provider immediately. 2. Elevate the head of the client's bed. 3. Document this as a normal and expected finding. 4. Administer morphine intravenously.

1. An S3 indicates left ventricular failure and should be reported to the health-care provider. It is a potential life-threatening complication of a myocardial infarction.

The nurse is teaching a patient about medications used to treat peripheral arterial disease and claudication. Which of these would the nurse include in the teaching plan? Select all that apply. Aspirin 1. Cholestyramine (Questran) 2. Cilostazol (Pletal) 3. Clopidogrel (Plavix) 4. Enoxaparin (Lovenox) 5. Ranolazine (Ranexa)

1. Cholestyramine (Questran) 3. Clopidogrel (Plavix) 4. Enoxaparin (Lovenox)

A client returns from a left heart catheterization. The right groin was used for catheter access. In which location should the nurse palpate the distal pulse on this client? 1. anterior to the right tibia 2. dorsal surface of the right foot 3. posterior to the right knee 4. right mid inguinal area

2. To best monitor that the client's circulation remains intact, the dorsal surface of the right foot should be palpated. When the left side of the heart is catheterized, the cannula enters via an artery. In this instance, the right femoral artery was accessed. While all options assess arterial points of the right leg, the dorsal surface of the right foot (the pedal pulse) is the most distal. If this pulse point is present and unchanged from before the procedure, the other pulse points should also be intact

A client asks the nurse about the difference between stable and unstable angina. Which statement by the nurse is most appropriate? 1. "Stable angina is unpredictable and lasts longer." 2. "Unstable angina is relieved by rest and medication." 3. "Stable angina can occur at rest or during sleep." 4. "Unstable angina occurs with moderate exertion."

Answer: 4 Rationales Option 1: Unstable angina is unpredictable and lasts longer. Option 2: Stable angina is relieved by rest and medication. Option 3: Unstable angina occurs at rest or during sleep. Option 4: This is a true statement.

The nurse would evaluate the patient as understanding teaching for prevention of coronary artery disease if the patient stated that which of the following is a risk factor for coronary artery disease that can be controlled? 1. "Family history of cardiovascular disease." 2. "Hypertension." 3. "Ethnicity." 4. "Family history of diabetes mellitus."

2. "Hypertension."

The nurse would evaluate the patient as understanding teaching on the purpose of coronary artery bypass graft surgery if the patient made which of the following litatements? 1. "It cures coronary artery disease." 2. "It is done to increase blood flow to the myocardium." 3. "It prevents spasms of the coronary arteries." 4. "It will decrease blood flow to the coronary arteries."

2. "It is done to increase blood flow to the myocardium."

The nurse has completed an assessment on a client with a decreased cardiac output. Which findings should receive the highest priority? 1. BP 110/62 mm Hg, atrial fibrillation with HR 82, bilateral basilar crackles 2. confusion, urine output 15 mL over the last 2 hours, orthopnea 3. SpO2 92 on 2 L nasal cannula, respirations 20, 1+ edema of lower extremities 4. weight gain of 1 kg in 3 days, BP 130/80, mild dyspnea with exercise

2. A low urine output and confusion are signs of decreased tissue perfusion. Orthopnea is a sign of left-sided heart failure. Crackles, edema, and weight gain should be monitored closely, but the levels are not as high a priority. With atrial fibrillation, there is a loss of atrial kick, but the blood pressure and heart rate are stable.

Which food should the nurse teach a client with heart failure to limit when following a 2-g sodium diet? 1. apples 2. canned tomato juice 3. whole wheat bread 4. beef tenderloin

2. Canned foods and juices such as tomato juice are typically high in sodium and should be avoided in a sodium-restricted diet. Canned foods and juices in which sodium has been removed or limited are available. The client should be taught to read labels carefully. Apples and whole wheat breads are not high in sodium. Beef tenderloin would have less sodium than canned foods or tomato juice.

The nurse is caring for a patient with cardiomyopathy. Which of the following symptoms, if reported by the patient, require priority action by the nurse? Select all that apply. 1. Decreased appetite 2. Dyspnea 3. Fatigue 4. Headache 5. Left great toe pain

2. Dyspnea; 3. Fatigue

.Which is the most appropriate diet for a client during the acute phase of myocardial infarction? 1. liquids as desired 2. small, easily digested meals 3. three regular meals per day 4. nothing by mouth

2. Recommended dietary principles in the acute phase of MI include avoiding large meals because small, easily digested foods are better tolerated. Fluids are given according to the client's needs, and sodium restrictions may be prescribed, especially for clients with manifestations of heart failure. Cholesterol restrictions may be prescribed as well. Clients are not prescribed diets of liquids only or restricted to nothing by mouth unless their condition is very unstable.

The nurse is caring for a patient recovering from a coronary angioplasty with stent placement. Which intervention is a priority for the patient at this time? 1. Securing chest tubes to bedding 2. Maintaining leg extension on the affected side 3. Discontinuing intravenous lines when taking oral fluids 4. Treating chest pain with intravenous morphine as needed

2. The cardiac catheter used to insert the stent is usually inserted via the femoral artery, a large, high-pressure vessel. The leg is maintained in extension for a prescribed period after the procedure to reduce the risk of bleeding, hematoma formation, or clot formation at the insertion site.

The client is scheduled for a right femoral cardiac catheterization. Which nursing intervention should the nurse implement after the procedure? 1. Perform passive range-of-motion exercises. 2. Assess the client's neurovascular status. 3. Keep the client in high Fowler's position. 4. Assess the gag reflex prior to feeding the client.

2. The nurse must make sure that blood is circulating to the right leg, so the client should be assessed for pulses, paresthesia, paralysis, coldness, and pallor.

ACE 1 inhibitors

Antihypertensive

Hydralazine (Apresoline)

Antihypertensive

valsartin (Diovan)

Antihypertensive

The nurse is evaluating patient teaching for mitral valve prolapse. The patient shows understanding of the prognosis by stating which of the following? 1. "The prognosis is poor." 2. "Heart failure often occurs." 3. "There are often no symptoms." 4. "Symptoms quickly progress."

3. "There are often no symptoms."

Which is the most important initial postprocedure nursing assessment for a client who has had a cardiac catheterization? 1. Monitor the laboratory values. 2. Observe neurologic function every 15 minutes. 3. Observe the puncture site for swelling and bleeding. 4. Monitor skin warmth and turgor.

3. Assessment of circulatory status, including observation of the puncture site, is of primary importance after a cardiac catheterization. Laboratory values and skin warmth and turgor are important to monitor but are not the most important initial nursing assessment. Neurologic assessment every 15 minutes is not required

The client has just returned from a cardiac catheterization. Which assessment data would warrant immediate intervention from the nurse? 1. The client's BP is 110/70 and pulse is 90. 2. The client's groin dressing is dry and intact. 3. The client refuses to keep the leg straight. 4. The client denies any numbness and tingling.

3. If the client bends the leg, it could cause the insertion site to bleed. This is arterial blood and the client could bleed to death very quickly, so this requires immediate intervention.

A client has risk factors for coronary artery disease, including smoking cigarettes, eating a diet high in saturated fat, and leading a sedentary lifestyle. The nurse can coach this client to improve health by: 1. explaining how the risk factors lead to poor health. 2. withholding praise until the client changes the risky behavior. 3. helping the client establish a wellness vision to reduce the health risks. 4. instilling mild fear into the client about the potential outcomes of the risky health behaviors.

3. In health coaching, unlike traditional client education techniques in which the nurse provides information, the goal of coaching is to encourage the client to explore the reasons for the behavior and establish a vision for health behavior and the way he or she can make changes to improve their health behavior and reduce or eliminate health risks. When coaching a client, the nurse does not provide information, withhold praise, or instill fear.

The nurse would evaluate the patient as understanding teaching for peripheral arterial occlusive disease if the patient stated that which of the following is the classic symptom? 1. Angina 2. Edema 3. Intermittent claudication 4. Stasis ulcers

3. Intermittent claudication

The nurse evaluates the patient as understanding how to prevent rheumatic fever if the patient identifies that rheumatic fever can be prevented by treating streptococcal infections with which of the following? 1. Cortisone 2. Cyclosporine 3. Penicillin 4. Prednisone

3. Penicillin

After the nurse reviews medication orders and patient allergies to give warfarin (Coumadin), which of the following actions should the nurse take first? 1. Obtain a glass of water. 2. Prepare the medication for administration. 3. Review international normalized ratio result. 4. Document the medication administration.

3. Review international normalized ratio result.

Which cardiac enzyme would the nurse expect to elevate first in a client diagnosed with a myocardial infarction (MI)? 1. Creatine kinase (CK-MB). 2. Lactate dehydrogenase (LDH). 3. Troponin. 4. White blood cells (WBCs).

3. Troponin is the enzyme that elevates within 1 to 2 hours.

normal potassium range

3.5-5.0 mEq/L

A client is admitted with chest pain and kept overnight for stress testing the next morning. Prior to sending the client to the stress test, the nurse reviews the results of the laboratory reports (see lab report). The nurse should report which elevated laboratory value to the healthcare provider (HCP) prior to the stress test? 1. cholesterol level 2. erythrocyte sedimentation rate 3. prothrombin time 4. troponin

4. The elevated troponin level should be reported to the HCP prior to the stress test as this change indicates myocardial damage. Sending the client to walk on a treadmill for stress testing would be contraindicated with evidence of recent myocardial injury and could further extend the damage. The other blood levels are helpful but not critical to this client's welfare at this point in time

regurgitation

A backward flowing, as in the backflow of blood through a defective heart valve.

pericardial effusion

A buildup of fluid in the pericardial space.

plaque

A deposit of fatty material on the lining of an artery.

autograft

A graft of tissue from one part of an individual's body to another part of the same individual's body.

Lisinopril

ACE inhibitor

Losartan (Cozaar)

ARB; antihypertension

Rhythm: none Heart rate: none P waves: none PRI: none QRS interval: none

ASYSTOLE RULES.

murmur

An abnormal sound heard on auscultation of the heart and adjacent large blood vessels.

A client with flutter waves on a recent electrocardiogram (ECG) reports syncopal episode, dyspnea, and angina. Which treatment should the nurse expect to be prescribed? 1. Synchronized cardioversion 2. Intravenous (IV) diltiazem 3. Transcutaneous pacing 4. Immediate defibrillation

Ans: 1 Rationales Option 1: Flutter waves on the ECG tracing are indicative of atrial flutter. The client is symptomatic, so synchronized cardioversion should be used to immediately convert the rhythm to normal sinus rhythm. Option 2: IV diltiazem, a calcium channel blocker, may be prescribed to control the ventricular rate if the client is asymptomatic. Option 3: Transcutaneous pacing is not indicated for atrial flutter. Option 4: Immediate defibrillation is not indicated for atrial flutter. Test Taking Tip: The client's condition is not stable, so the nurse should identify the treatment that will most effectively treat the client in the shortest period of time.

Rhythm: depends on the underlying rhythm; PVC usually interrupts rhythm Heart rate: depends on underlying rhythm P waves: absent before PVC QRS complex PRI: none for PVC QRS interval: if PVC, is greater than 0.10 seconds; T wave is in the opposite direction of QRS complex (i.e., QRS upright, T downward; or QRS downward, T upright).

PREMATURE VENTRICULAR CONTRACTION RULES.

The nurse is preparing to administer digoxin (Lanoxin). Which of the following interventions should the nurse implement? 1. Monitor apical heart rate for 1 minute. 2. Assess for symptoms of hyperkalemia. 3. Take the client's blood pressure. 4. Weight the client.

Ans: 1 The nurse is preparing to administer digoxin (Lanoxin). Which of the following interventions should the nurse implement? 1. Monitor apical heart rate for 1 minute. 2. Assess for symptoms of hyperkalemia. 3. Take the client's blood pressure. 4. Weight the client.

The nurse is caring for a male client with angina. Which clinical manifestations can the nurse expect to find? Select all that apply. 1. Heaviness in chest 2. Dyspnea 3. Fatigue 4. Diaphoresis 5. Radiating jaw pain

Ans: 1, 2, 4, 5 Rationales Option 1: Heaviness in the chest is common in men experiencing a myocardial infarction (MI). Option 2: Dyspnea is common in men experiencing an MI. Option 3: This is typical in women experiencing an MI. Option 4: This is seen in men experiencing an MI. Option 5: This is commonly seen in men having an MI. Test Taking Tip: Women do not typically seek treatment for symptoms of an MI, which are different from those in men.

The nurse is caring for a client with a myocardial infarction (MI). Which of the following are used as treatments for an MI? Select all that apply. 1. Morphine 2. Hydrocodone 3. Oxygen 4. Nitroglycerin 5. Aspirin

Ans: 1, 3, 4, 5 Rationales Option 1: Morphine is used to treat an MI. Option 2: Hydrocodone is not used to treat an MI. Option 3: Oxygen is used for an MI. Option 4: Nitro is used to treat an MI. Option 5: Aspirin is used in the treatment of an MI. Test Taking Tip: MONA is an acronym for the treatment of an MI.

The nurse is caring for a client with an arterial thrombosis. Which of the following are contributing factors? Select all that apply. 1. Polycythemia 2. Hypothyroidism 3. Fluid volume excess 4. Repeated arterial needlesticks 5. Hypotension

Ans: 1, 4, Rationales Option 1: Polycythemia is a cause of arterial thrombosis. Option 2: Hypothyroidism is not a cause of arterial thrombosis. Option 3: Dehydration is a cause of arterial thrombosis. Option 4: Arterial needlesticks are a contributing factor to an arterial thrombosis. Option 5: Hypotension is not a cause of arterial thrombosis.

The nurse is caring for a client with infective endocarditis (IE). Which clinical manifestations can the nurse expect to find? Select all that apply. 1. Petechiae 2. Pericardial effusion 3. Pericardial friction rub 4. Aching muscles 5. Fever

Ans: 1, 4, 5 Rationales Option 1: Petechiae is found in clients with infective endocarditis. Option 2: A pericardial effusion is found in clients with pericarditis. Option 3: A pericardial friction rub is found in clients with pericarditis. Option 4: Aching muscles are a common finding in clients with IE. Option 5: Fever is a common finding in clients with IE.

high-density lipoprotein

Plasma lipids bound to albumin consisting of lipoproteins. It has been found that those with high levels of HDL have less chance of having coronary artery disease.

valvuloplasty

Plastic or restorative surgery on a valve, especially a cardiac valve.

venous stasis ulcers

Poorly healing ulcers that result from inadequate venous drainage.

Dressler syndrome

Postmyocardial infarction syndrome; pericarditis. Also called Dressler's syndrome.

The nurse is teaching a colleague about blood flow through the heart. Which teaching should the nurse include? 1. Blood flows from the left atrium to the right atrium. 2. Blood flows from the right atrium through the tricuspid valve. 3. Blood flows from the left ventricle through the mitral valve. 4. Blood flows from the left ventricle to the right ventricle.

Ans: 2 Rationales Option 1: Blood flows from the left atrium to the left ventricle. Option 2: Blood flows from the right atrium and through the tricuspid valve (right atrioventricular [AV] valve) to the right ventricle. The tricuspid valve prevents backflow of blood. Option 3: Blood flows from the left atrium and through the mitral valve (left AV valve) to the left ventricle. Option 4: Blood flows from the left ventricle through the aortic semilunar valve to the aorta.

bioprosthesis

Prosthesis consisting of an animal part or containing animal tissue (e.g., porcine heart valve).

The nurse is assessing the temperature of a client's lower extremities and notes that one leg is cool to the touch. Which of these answers describes the meaning of a cool extremity? 1. Decreased arterial blood flow 2. Decreased venous blood flow 3. Fluid is pooling to the area 4. Fluid volume deficit

Rationales Option 1: In areas of decreased blood flow, the ischemic area feels cooler than the rest of the body because blood warms the body. Option 2: Decreased venous blood flow causes the extremity to be warm. Option 3: Fluid pooling causes edema of the extremities. Option 4: Fluid volume deficit does not cause cool extremities.

The nurse is caring for a group of clients. Which client is at highest risk for mitral valve prolapse (MVP)? 1. A 40-year-old male client 2. A 60-year-old female client 3. A 25-year-old male client 4. A 20-year-old female client

Rationales Option 1: MVP typically occurs in women age 15 to 30. Option 2: MVP typically occurs in women age 15 to 30. Option 3: This client is at risk due to age, but not gender. Option 4: This client is at high risk for MVP due to age and gender. Test Taking Tip: MVP is common in clients who are women age 15 to 30.

The nurse is caring for a female client experiencing a myocardial infarction (MI). Which clinical manifestation is the client likely to exhibit? 1. Arm pain 2. Shoulder pain 3. Heartburn 4. Back pain

Ans: 3 1. Arm pain 2. Shoulder pain 3. Heartburn 4. Back pain Rationales Option 1: Arm pain is more common in men. Option 2: Shoulder pain is more common in men. Option 3: Heartburn is more common in women. Option 4: Back pain is more common in men. Test Taking Tip: Women have atypical symptoms for an MI and often do not seek treatment.

The nurse is preparing a presentation about preventing coronary artery disease (CAD). Which of the following should the nurse include in the teaching? 1. Increasing blood glucose 2. Reducing exercise 3. Smoking cessation 4. Working a high-stress job

Ans: 3 Rationales Option 1: Blood glucose should be reduced to reduce the risk of CAD. Option 2: Exercise should be increased. Option 3: Effects of smoking cessation are immediate and can reduce the risk for CAD. Option 4: Stress should be reduce

The nurse is caring for a client with lymphangitis. Which clinical manifestation can the nurse expect to find? 1. Loss of hair in the affected extremity 2. Spider veins 3. Red streaks on the extremity 4. Brownish extremities

Ans: 3 Rationales Option 1: Loss of hair is found in arterial occlusion. Option 2: Spider veins are common in varicose veins. Option 3: Red streaks on the extremity are indicative of lymphangitis. Option 4: This is not a finding of lymphangitis.

The nurse is caring for a client with myocarditis. Which of the following interventions should the nurse implement? 1. Instruct the client to minimize alcohol intake. 2. Encourage exercise. 3. Administer lisinopril as ordered. 4. Plan activities all at once.

Ans: 3 Rationales Option 1: The client should avoid alcohol entirely. Option 2: The client will be on bed rest with limited activity. Option 3: The nurse should administer ACE inhibitors or beta blockers. Option 4: Activities should be spaced out with periods of rest.

A client scheduled for an exercise stress test states, "I am not able to exercise." The nurse should assess the client for an allergy to which medication? 1. Amiodarone 2. Aspirin 3. Adenosine 4. Amlodipine

Ans: 3 Rationales Option 1: There is no reason to assess for an allergy to amiodarone. Option 2: There is no reason to assess for an allergy to aspirin. Option 3: If the client is unable to exercise, the nurse should anticipate that the health-care provider will order a medication stress test. During this test, a coronary vasodilator (adenosine or dipyridamole) is given to increase blood flow in healthy vessels and reveal unhealthy areas with reduced blood flow. Option 4: There is no reason to assess for an allergy to amlodipine.

The nurse is caring for a client who underwent abdominal aortic aneurysm (AAA) repair. Which of the following should concern the nurse? 1. Urine output of 50 mL/hr 2. Report of level 3 pain on a 0-10 scale 3. Increase in abdominal girth measurement 4. Pulses equal bilaterally

Ans: 3 Rationales Option 1: This is normal urine output. Option 2: This is typical after surgery. Option 3: This can be indicative of hemorrhage. Option 4: This is not concerning.

The nurse is providing teaching to a client about risk factors for heart disease. Which of the following is a nonmodifiable risk factor? 1. Alcohol use 2. Obesity 3. Hypertension 4. Ethnicity

Ans: 4 Rationales Option 1: Alcohol use is a modifiable risk factor. Option 2: Obesity is a modifiable risk factor. Option 3: Hypertension is a modifiable risk factor. Option 4: Ethnicity is a nonmodifiable risk factor. Test Taking Tip: Modifiable risk factors can be changed, whereas nonmodifiable risk factors cannot.

Which internal factor may contribute to hypertension? 1. Decreased blood viscosity 2. Decreased fluid volume 3. Decreased peripheral vascular resistance 4. Decreased elasticity of blood vessels

Ans: 4 Rationales Option 1: An increase in blood viscosity leads to hypertension. Option 2: An increase in fluid volume may lead to hypertension. Option 3: An increase in peripheral vascular resistance (PVR) contributes to hypertension. Option 4: This is a contributing factor to hypertension.

Rhythm: regular Heart rate: 101 to 180 bpm P waves: rounded, upright, precede each QRS complex, alike PRI: 0.12 to 0.20 seconds QRS interval: less than or equal to 0.10 seconds

SINUS TACHYCARDIA RULES

angina pectoris

Severe pain and pressure in the chest caused by insufficient supply of blood and oxygenation to the heart.

emboli

Solid, liquid, or gaseous masses of undissolved matter traveling with the fluid current in a blood or lymphatic vessel.

pericardiocentesis

Surgical perforation of the pericardium.

varicose veins

Swollen, distended, and knotted veins, usually in the subcutaneous tissue of the leg.

arteriosclerosis

Term applied to a number of pathological conditions in which there is gradual thickening, hardening, and loss of elasticity of the walls of the arteries.

insufficiency

The condition of being inadequate for a given purpose, such as heart valves that do not close properly.

Rhythm: chaotic and extremely irregular Heart rate: not measurable P waves: none PRI: none QRS complex: no

VENTRICULAR FIBRILLATION RULES

The nurse is reviewing a client's electrocardiogram (ECG) tracing. Which of the following are characteristics of normal sinus rhythm (NSR)? 1. PR interval 0.34 to 0.43 seconds 2. Heart rate 50-100 bpm 3. QRS interval less than or equal to 0.10 seconds 4. P waves are rounded and inverted, and follow each QRS complex

Ans: Option 3 Rationales Option 1: The normal PR interval is 0.12 to 0.20 seconds. The normal QT interval is 0.34-0.43 seconds. Option 2: NSR heart rate is between 60 and 100 bpm. Option 3: The normal QRS interval is less than or equal to 0.10 seconds. Option 4: P waves are rounded and upright: they precede each QRS complex; and they are all alike in NSR.

Carvedilol (Coreg)

Beta-blocker (antihypertension)

Lasix (furosemide)

Classification: Loop Diuretic

ischemia

Condition of inadequate blood supply.

myocardial infarction

Death of cells of an area of the myocardium (heart muscle) as a result of oxygen deprivation, which in turn is caused by obstruction of the blood supply. Commonly referred to as a heart attack.

hyperlipidemia

Excessive quantity of fat in the blood.

pericardiectomy

Excision of part or all of the pericardium.

endarterectomy

Excision of thickened atheromatous areas of the innermost coat of an artery.

Calcium Channel Blockers

agents that inhibit the entry of calcium ions into heart muscle cells, causing a slowing of the heart rate, a lessening of the demand for oxygen and nutrients, and a relaxing of the smooth muscle cells of the blood vessels to cause dilation; used to prevent or treat angina pectoris, some arrhythmias, and hypertension

metoprolol

beta blocker

Beta Blockers

decrease heart rate and dilate arteries by blocking beta receptors

cardiomegaly

enlargement of the heart

Normal QRS interval

less than or equal to 0.10 seconds. 1 - 3 small boxes.

diuretics

medications administered to increase urine secretion in order to rid the body of excess water and salt

Question 2. A client scheduled for an exercise stress test states, "I am not able to exercise." The nurse should assess the client for an allergy to which medication? 1. Amiodarone 2. Aspirin 3. Adenosine 4. Amlodipine

option 3 Rationales Option 1: There is no reason to assess for an allergy to amiodarone. Option 2: There is no reason to assess for an allergy to aspirin. Option 3: If the client is unable to exercise, the nurse should anticipate that the health-care provider will order a medication stress test. During this test, a coronary vasodilator (adenosine or dipyridamole) is given to increase blood flow in healthy vessels and reveal unhealthy areas with reduced blood flow. Option 4: There is no reason to assess for an allergy to amlodipine.

Spironolactone (Aldactone)

potassium sparing diuretic

HCTZ (hydrochlorothiazide)

thiazide diuretic

normal magnesium range

1.5-2.5mEq/L

Normal heart rate

60-100 bpm

The nurse reviews laboratory data for a client admitted to the emergency department with chest pain. Which serum value requires the most immediate action by the nurse? 1. Glucose 200 mg/dL (11.1 mmol/L) 2. Hematocrit 38% (0.38) 3. Potassium 3.4 mEq/L (3.4 mmol/L) 4. Troponin 0.7 ng/mL (0.7 mcg/L)

ANS : 4 Troponin is a highly specific cardiac marker for the detection of MI. It has greater sensitivity and specificity for myocardial injury than creatine kinase (CK) MB

Which hormones are secreted by the atria? 1. Atrial natriuretic peptide (ANP) 2. Aldosterone 3. Progesterone 4. Gonadotropin-releasing hormone

Ans: 1 Rationales Option 1: ANP is secreted by the atria. Option 2: Aldosterone is not secreted by the atria. Option 3: Progesterone is not secreted by the atria. Option 4: This is not secreted by the atria.

In heart failure, which part of the heart fails first? 1. Left atrium 2. Left ventricle 3. Right atrium 4. Right ventricle

Ans: 2 Rationales Option 1: The left ventricle fails first. Option 2: The left ventricle fails first because the left ventricle has the greatest workload. Option 3: The left ventricle fails first. Option 4: Once the left side fails, the right one will fail from having to do the work of the left ventricle.

Which of the following are valves of the heart? Select all that apply. 1. Atrium 2. Mitral 3. Tricuspid 4. Pulmonic 5. Aortic

Ans: 2, 3, 4, 5 Rationales Option 1: The atrium is not a valve, but a chamber. Option 2: Mitral is a valve. Option 3: Tricuspid is a valve. Option 4: Pulmonic is a valve. Option 5: Aortic is a valve. Test Taking Tip: The four chambers of the heart include the left and right atrium and the left and right ventricles.

The nurse is providing teaching to a client with Buerger disease. Which of the following statements made by the client indicates a need for further teaching? 1. "I can take clopidogrel (Plavix), and it may help a little." 2. "If my finger turns gangrenous, it may be amputated." 3. 'My fingers may be pale when it is cold." 4. "I can still smoke as long as I cut back."

Ans: 3 Rationales Option 1: This statement is correct; antiplatelet drugs are used for treatment. Option 2: This is an accurate statement. Option 3: This is an accurate statement. Option 4: This requires correction. The client needs to avoid all forms of tobacco.

The nurse is providing teaching for a client with mitral valve prolapse. Which of the following statements made by the client indicates an understanding of the teaching? 1. "I should avoid exercise because it will put strain on my heart." 2. "I can have caffeine if I have only three sodas per day." 3. "I should avoid taking aspirin." 4. "I should follow a healthy diet."

Ans: 4 Rationales Option 1: The client should be encouraged to exercise. Option 2: Caffeine should be avoided. Option 3: Aspirin should be encouraged to prevent formation of blood clots on the valve. Option 4: The client should follow a good diet.

valvotomy

Cutting through a valve

Enalapril (Vasotec)

*class*: ACE Inhibitor, antihypertensives *Indication*: hypertension, management of CHF *Nursing Considerations*: - can cause neutropenia - check WBCs regularly - use cautiously with potassium supplements and potassium sparing diuretics. - use cautiously with diuretic therapy - administer 1 hour before meals - monitor blood pressure often - monitor weight and fluid status - monitor renal profile - monitor CBC frequently - dry cough

Diltiazem (Cardizem)

*class*: Ca Channel Blocker (Bezothiazepine), antianginals, antiarrhythmics, antihypertensive *Indication*: hypertension, angina, SVT, a-fib, aflutter *Nursing Considerations*: - contraindicated in 2nd and 3rd AV block - may cause arrhythmias, CHF, bradycardia, peripheral edema, gingival hyperplasia - increases digoxin levels - don't drink *grapefruit juice* - assess for signs of CHF - monitor EKG continuously - tell patient to change positions slowly - monitor serum potassium - instruct pt on how to take blood pressure

The nurse is caring for a client who has just returned from having a percutaneous transluminal balloon angioplasty with femoral artery access. In which order, from first to last, should the nurse obtain information about the client? All options must be used. 1. vital signs and oxygen saturation 2. pedal pulses 3. color and sensation of extremity 4. catheterization site

1,2,4,3. When a client returns from having a transluminal balloon angioplasty with femoral access, the nurse should first obtain baseline vital signs and oxygen saturation to determine evidence of bleeding or decreased tissue perfusion. The nurse should next assess the pedal pulses to determine if the client has adequate peripheral tissue perfusion. Next the nurse should inspect the catheterization site and then determine color and sensation in the affected leg.

The nurse is evaluating understanding after a teaching session for mechanical cardiac valve replacement surgery. Which statement by the patient indicates understanding of teaching? 1. "I will need anticoagulants for the first month after surgery." 2. "I will need anticoagulant therapy for life." 3. "I will need anticoagulant therapy for the first year after valve replacement." 4. "I will not need to be on anticoagulant therapy."

2. "I will need anticoagulant therapy for life."

The nurse is caring for a group of patients. After completing morning rounds, which of the following patients require priority care? 1. A patient who is 2 days postsurgery reporting severe constipation 2. A patient with a deep vein thrombosis who has peripheral edema 3. A patient with aortic stenosis who is reporting chest pain 4. A patient with mitral valve prolapse who has lost 2 pounds of weight this morning

3. A patient with aortic stenosis who is reporting chest pain

The client diagnosed with a myocardial infarction is six (6) hours post-right femoral percutaneous transluminal coronary angioplasty (PTCA), also known as balloon surgery. Which assessment data would require immediate intervention by the nurse? 1. The client is keeping the affected extremity straight. 2. The pressure dressing to the right femoral area is intact. 3. The client is complaining of numbness in the right foot. 4. The client's right pedal pulse is 3+ and bounding.

3. Any neurovascular assessment data that are abnormal require intervention by the nurse; numbness may indicate decreased blood supply to the right foot.

The client who experiences angina has been told to follow a low cholesterol diet. Which meal would be best? 1. hamburger, salad, and milk shake 2. baked liver, green beans, and coffee 3. spaghetti with tomato sauce, salad, and coffee 4. fried chicken, green beans, and skim milk

3. Pasta, tomato sauce, salad, and coffee would be the best selection for the client following a low-cholesterol diet. Hamburgers, milk shakes, liver, and fried foods tend to be high in cholesterol.

normal glucose range

65-99 mg/dL

normal calcium range

8.5-10.5 mg/dL

atherosclerosis

A form of arteriosclerosis characterized by accumulation of plaque, blood, and blood products lining the wall of the artery, causing partial or complete blockage of an artery.

cardiomyopathy

A group of diseases that affect the myocardium's (heart muscle's) structure or function.

rheumatic fever

A hypersensitivity reaction to antigens of group A beta-hemolytic streptococci.

Rhythm: atrial rhythm regular; ventricular rhythm regular or irregular depending on consistency of AV conduction of impulses Heart rate: ventricular rate varies P waves: flutter or F waves with sawtooth pattern PRI: none measurable QRS interval: less than or equal to 0.10 seconds

ATRIAL FLUTTER RULES.

The nurse is reviewing orders for a client with peripheral arterial disease (PAD). Which of the following medications should the nurse anticipate administering? 1. Nitroglycerin (Nitrostat) 2. Enalapril (Vasotec) 3. Cilostazol (Pletal) 4. Amlodipine (Norvasc)

Ans: 3 Rationales Option 1: This is used for angina. Option 2: Vasotec is not given for PAD. Option 3: This medication is an antiplatelet drug used to treat claudication. Option 4: Norvasc is not used for PAD.

The nurse is caring for a client who underwent abdominal aortic aneurysm (AAA) repair. Which of the following should concern the nurse? 1. Urine output of 50 mL/hr 2. Report of level 3 pain on a 0-10 scale 3. Increase in abdominal girth measurement 4. Pulses equal bilaterally

Ans: Option 3 Rationales Option 1: This is normal urine output. Option 2: This is typical after surgery. Option 3: This can be indicative of hemorrhage. Option 4: This is not concerning.

The nurse is caring for a client on warfarin. A normal prothrombin time (PT) range is 9 to 12 seconds. Calculate the therapeutic range. Enter the numeral only.

Correct Answer - 13.5 to 24 A normal therapeutic range is 9 to 12 seconds. Warfarin's therapeutic range is 1.5 to 2 times the normal PT range. Multiply 9 by 1.5 to equal 13.5 for the low end of the range. Multiple 12 by 2 to equals 24 for the high end of the range.

embolism

Foreign substance or blood clot that travels through the circulatory system until it obstructs a vessel.

thrombosis

Formation, development, or presence of a thrombus, an aggregation of blood factors.

heterograft

Graft from one species to another species (animal to man). Another name for xenograft.

beta-hemolytic streptococci

Gram-positive bacteria that, when grown on blood-agar plates, completely hemolyze the blood and produce a clear zone around the bacteria colony. Group A beta-hemolytic streptococci cause disease in humans.

infective endocarditis

Inflammation of the heart lining caused by microorganisms.

pericarditis

Inflammation of the pericardium (two thin layers of a saclike membrane surrounding the heart).

The nurse is assigned to care for four clients. Which client should the nurse see first? 1. An older adult client whose systolic blood pressure (BP) dropped 12 mm Hg after position changes 2. A younger adult client who has an S3 heart sound 3. An older adult client who has an S4 heart sound 4. A younger adult client whose systolic to diastolic to pulse pressure ratio is 3:2:1

Option 3 Rationales Option 1: A systolic BP drop of less than 15 mm Hg with position changes is normal. This client is stable. Option 2: Auscultation of an S3 heart sound in a younger adult is a normal finding. This client is stable. Option 3: Auscultation of an S4 heart sound in an older adult client can occur with hypertension, coronary artery disease, and pulmonary stenosis. This is the only client whose condition is potentially unstable. Option 4: The usual ratio of systolic to diastolic to pulse pressure is 3:2:1. This client is stable. Test Taking Tip: The client that the nurse should see first is the least stable client who has the highest likelihood of a real or potential medical emergency.

Rhythm: premature beat interrupts underlying rhythm where it occurs Heart rate: depends on the underlying rhythm; if NSR, 60 to 100 bpm P waves: early beat is abnormally shaped PRI: usually appears normal, but premature beat could have shortened or prolonged PRI QRS interval: less than or equal to 0.10 seconds (indicates normal conduction to ventricles)

PREMATURE ATRIAL CONTRACTIONS RULEs

A client with a diagnosis of hypertension who has been taking medication reports a dry cough. Which medication can the nurse expect to find in the client's medication history? 1. Amlodipine 2. Lisinopril 3. Clonidine 4. Furosemid

Rationales Option 1: Calcium channel blockers do not cause dry cough. Option 2: The side effect of an ACE inhibitor is dry cough. Option 3: Central-acting alpha2 agonists do not cause dry cough. Option 4: Loop diuretics do not cause dry cough. Test Taking Tip: ACE inhibitors, also known as "Prils," can cause a continuous cough.

The nurse is caring for a group of clients. Which client is at highest risk for developing Buerger disease? 1. A client with hepatitis who drinks often 2. A client with anorexia who occasionally smokes 3. A client with peptic ulcer disease who drinks socially 4. A client with lupus who is a heavy smoke

Rationales Option 1: This client is not at high risk for Buerger disease. Option 2: This client is slightly at risk (smoking). Option 3: This client is not at high risk for Buerger disease. Option 4: This client is at highest risk for Buerger disease (autoimmune disease and heavy smoker). Test Taking Tip: All use of tobacco must be avoided.

Which of the following is a nonmodifiable risk factor for heart disease? 1. Age 2. Tobacco use 3. Obesity 4. Lack of exercise

Rationales Option 1: This is a non-modifiable risk factor. Option 2: This is a modifiable risk factor. Option 3: This is a modifiable risk factor. Option 4: This is a modifiable risk factor. Test Taking Tip: Non-modifiable means cannot be changed.

petechiae

Small, purplish, hemorrhagic spots on the skin that appear in certain illnesses and bleeding disorders.

commissurotomy

Surgical incision of any commissure, as in cardiac valves to increase the size of the orifice.

myectomy

Surgical removal of a hypertrophied muscle.

Rhythm: P-to-P interval regular; R-to-R interval regular; atria and ventricles controlled by separate electrical impulses from foci that are firing regularly Heart rate: atrial 60 to 100 bpm; ventricular rate slower: 40 to 60 bpm is junctional foci; 20 to 40 bpm is ventricular foci P waves: rounded, upright, alike; more P waves than QRS complexes; may occur within a QRS complex or upon a T wave PRI: no P waves conducted to the ventricles, so no relationship to the QRS complexes; therefore, there is no actual PRI (may appear as if the PRI varies) QRS interval: less than or equal to 0.10 (junctional origin); greater than 0.10 (ventricular origin)

THIRD-DEGREE ATRIOVENTRICULAR BLOCK RULES.

stenosis

The constriction or narrowing of a passage or orifice, such as a cardiac valve.

thrombophlebitis

The formation of a clot and inflammation within a vein.

anastomosed

To surgically connect two parts.

allograft

Transplanted organ, tissue, or cells from one individual to another of the same species who is not genetically identical. Donors can be living (related or unrelated) or cadaveric.

Rhythm: usually regular, may have some irregularity Heart rate: 150 to 250 ventricular bpm; slow VT is below 150 bpm P waves: absent PRI: none QRS interval: greater than 0.10 seconds

VENTRICULAR TACHYCARDIA RULES.

annuloplasty

repair of a cardiac valve

collateral circulation

Small branches off of larger blood vessels that will increase in size and capacity next to a main blood vessel that is obstructed.

international normalized ratio

The World Health Organization's standard for reporting the prothrombin time assay test when the thromboplastin reagent developed by the first International Reference Preparation is used. The reagent was developed to prevent variability in prothrombin time testing results and provide uniformity in monitoring therapeutic levels for coagulation during oral anticoagulation therapy.

Atenolol (Tenormin)

*class*: beta blocker, antianginal, antihypertensive *Indication*: hypertension, angina, prevention of MI - May cause bradycardia, CHF, pulmonary edema - Masks symptoms associated with diabetes mellitus - advise to change positions slowly to prevent orthostatic hypotension - instruct patient on how to take blood pressure

A client is admitted with a myocardial infarction and atrial fibrillation. While auscultating the heart, the nurse notes an irregular heart rate and hears an extra heart sound at the apex after the S2 that remains constant throughout the respiratory cycle. The nurse should document these findings as: 1. heart rate irregular with S3 . 2. heart rate irregular with S4 . 3. heart rate irregular with aortic regurgitation. 4. heart rate irregular with mitral stenosis.

1. An S3 heart sound occurs early in diastole as the mitral and tricuspid valves open and blood rushes into the ventricles. To distinguish an S3 from a physiologic S2 split, a split S2 occurs during inspiration and S3 remains constant during the respiratory cycle. Its pitch is softer and best heard with the bell at the apex, and it is one of the first clinical findings in left ventricular failure. An S4 is heard in late diastole when atrial contraction pumps volume into a stiff, noncompliant ventricle. An S4 is not heard in a client with atrial fibrillation because there is no atrial contraction. Murmurs are sounds created by turbulent blood flow through an incompetent or stenotic valve.

The nurse is assessing the client diagnosed with congestive heart failure. Which laboratory data would indicate that the client is in severe congestive heart failure? 1. An elevated B-type natriuretic peptide (BNP). 2. An elevated creatine kinase (CK-MB). 3. A positive D-dimer. 4. A positive ventilation/perfusion (V/Q) scan

1. BNP is a specific diagnostic test. Levels higher than normal indicate congestive heart failure, with the higher the number, the more severe the CHF.

The nurse is collecting data on a patient who had surgery. Which of the following signs and symptoms indicate to the nurse the possible presence of a deep vein thrombosis in the patient's leg that should be reported to the health care provider? Select all that apply. 1. Calf swelling 2. Crackles 3. Jugular venous distention 4. Fever 5. Warmth 6. Redness

1. Calf swelling; 4. Fever; 5. Warmth; 6. Redness;

The licensed practical nurse is assisting with collecting data on a female patient. Which of these findings should be reported to the registered nurse that could be possible symptoms of a myocardial infarction in the absence of chest pain? Select all that apply. 1. Fatigue 2. Dizziness 3. Nausea 4. Pain between shoulder blades 5. Sweating 6. Shortness of breath

1. Fatigue 3. Nausea 4. Pain between shoulder blades 6. Shortness of breath

A 68-year-old client on day 2 after hip surgery has no cardiac history but reports having chest heaviness. The nurse should first: 1. inquire about the onset, duration, severity, and precipitating factors of the heaviness. 2. administer oxygen via nasal cannula. 3. offer pain medication for the chest heaviness. 4. inform the healthcare provider (HCP) of the chest heaviness.

1. Further assessment is needed in this situation. It is premature to initiate other actions until further data have been gathered. Inquiring about the onset, duration, location, severity, and precipitating factors of the chest heaviness will provide pertinent information to convey to the HCP

An older adult with a history of heart failure is admitted to the emergency department with pulmonary edema. On admission, what should the nurse assess first? 1. blood pressure 2. skin breakdown 3. serum potassium level 4. urine output

1. It is a priority to assess blood pressure first because people with pulmonary edema typically experience severe hypertension that requires early intervention. The client probably does not have skin breakdown, but when the client is stable and when the nurse obtains a complete health history, the nurse should inspect the client's skin for any signs of breakdown; however, when the client is stable, the nurse should inspect the skin. Potassium levels are not the first priority. The nurse should monitor urine output after the client is stable

What actions can the nurse take to reduce the anxiety of a patient who is experiencing chest pain? Select all that apply. 1. Remain with the patient at all times. 2. Dim lights, close door, and leave patient to sleep. 3. Explain heart's function is being monitored. 4. Explain procedures and actions taken. 5. Turn television on for distraction. 6. Allow family to be involved in care.

1. Remain with the patient at all times. 3. Explain heart's function is being monitored. 4. Explain procedures and actions taken. 6. Allow family to be involved in care.

The nurse is caring for a client diagnosed with an anterior myocardial infarction 2 days ago. Upon assessment, the nurse identifies a systolic murmur at the apex. The nurse should first: 1. assess for changes in vital signs. 2. draw an arterial blood gas. 3. evaluate heart sounds with the client leaning forward. 4. obtain a 12-lead electrocardiogram.

1. The nurse should first obtain vital signs as changes in the vital signs will reflect the severity of the sudden drop in cardiac output: decrease in blood pressure, increase in heart rate, and increase in respirations. Infarction of the papillary muscles is a potential complication of an MI causing ineffective closure of the mitral valve during systole. Mitral regurgitation results when the left ventricle contracts and blood flows backward into the left atrium, which is heard at the fifth intercostal space, left midclavicular line. The murmur worsens during expiration and in the supine or left-side position and can best be heard when the client is in these positions, not with the client leaning forward. A 12-lead ECG views the electrical activity of the heart; an echocardiogram views valve function.

When assessing an older adult, the nurse finds the apical impulse below the fifth intercostal space. The nurse should further assess the client for: 1. left atrial enlargement. 2. left ventricular enlargement. 3. right atrial enlargement. 4. right ventricular enlargement.

2. A normal apical impulse is found over the apex of the heart and is typically located and auscultated in the left fifth intercostal space in the midclavicular line. An apical impulse located or auscultated below the fifth intercostal space or lateral to the midclavicular line may indicate left ventricular enlargement.

A client with chest pain is prescribed intravenous nitroglycerin. Which assessment is of greatest concern for the nurse initiating the nitroglycerin drip? 1. Serum potassium is 3.5 mEq/L(3.5 mmol/L). 2. Blood pressure is 88/46 mm Hg. 3. ST elevation is present on the electrocardiogram. 4. Heart rate is 61 bpm

2. Nitroglycerin is a vasodilator that will lower blood pressure. The client is having chest pain, and the ST elevation indicates injury to the myocardium, which may benefit from nitroglycerin. The potassium and heart rate are within normal range

Rheumatic fever is a rare complication of which condition? 1. Influenza 2. Bronchitis 3. Strep throat 4. Laryngitis

Ans #3 Rationales Option 1: Rheumatic fever is not a complication of influenza. Option 2: Bronchitis does not cause rheumatic fever. Option 3: Rheumatic fever is a rare complication of strep throat. Option 4: Laryngitis does not cause rheumatic fever.

A client arrives in the emergency department complaining of chest pain that began 4 hours ago. A troponin T blood specimen is obtained, and the results indicate a level of 0.6 ng/mL. The nurse determines that this result indicates which finding? 1. A normal level 2. A low value that indicates possible gastritis 3. A level that indicates a myocardial infarction 4. A level that indicates the presence of possible angina

3 Rationale: Troponin is a regulatory protein found in striated muscle. The troponins function together in the contractile apparatus for striated muscle in skeletal muscle and in the myocardium. Increased amounts of troponins are released into the bloodstream when an infarction causes damage to the myocardium. A troponin T value that is higher than 0.1 to 0.2 ng/mL is consistent with a myocardial infarction. A normal troponin I level is lower than 0.6 ng/mL

The nurse notices that a client's heart rate decreases from 63 to 50 bpm on the monitor. The nurse should first: 1. administer atropine 0.5 mg IV push. 2. auscultate for abnormal heart sounds. 3. prepare for transcutaneous pacing. 4. take the client's blood pressure

4. The nurse should first assess the client's tolerance to the drop in heart rate by checking the blood pressure and level of consciousness and - determine if atropine is needed. If the client is symptomatic, atropine and transcutaneous pacing are interventions for symptomatic bradycardia. Once the client is stable, further physical assessments can be done

Which client is at greatest risk for coronary artery disease? 1. a 32-year-old female with mitral valve prolapse who quit smoking 10 years ago 2. a 43-year-old male with a family history of CAD and cholesterol level of 158 (8.8 mmol/L) 3. a 56-year-old male with an HDLof 60 (3.3 mmol/L) who takes atorvastatin 4. a 65-year-old female who is obese with an LDL of 188 (10.4 mmol/L)

4. The woman who is 65 years old, is overweight, and has an elevated LDLis at greatest risk. Total cholesterol >200 (11.1 mmol/L), LDL >100 (5.5 mmol/L), HDL<40 (2.2 mmol/L) in men, HDL<50 (2.8 mmol/L) in women, men 45 years and older, women 55 years and older, smoking, and obesity increase the risk of CAD. Atorvastatin reduces LDLand decreases risk of CAD. The combination of postmenopausal, obesity, and high LDL places this client at greatest risk.

The nurse is reinforcing teaching for a patient prescribed sublingual nitroglycerin tablets. The nurse should instruct the patient to use this medication in which of the following ways? 1. Take one tablet and lie down for 1 hour, and repeat if pain unrelieved. 2. Place two tablets under the tongue daily to prevent angina. 3. Swallow one tablet, wait 10 minutes; swallow two tablets if pain persists; swallow three tablets if pain remains after 15 minutes. 4. With angina and symptoms of myocardial infarction, place one tablet under the tongue and if, after 5 minutes has elapsed, the pain is unchanged or worse, call 911."

4. With angina and symptoms of myocardial infarction, place one tablet under the tongue and if, after 5 minutes has elapsed, the pain is unchanged or worse, call 911."

low-density lipoprotein (LDL)

A lipoprotein that transports cholesterol and triglycerides from the liver to peripheral tissues. LDL allows fats and cholesterol to move within the water-based solution of the blood. Increased LDL cholesterol is associated with cardiovascular disease, so it is often referred to as "bad cholesterol."

chorea

A nervous system condition marked by involuntary muscular twitching of the limbs or facial muscles.

Raynaud disease

A primary or idiopathic vasospastic disorder characterized by bilateral and symmetrical pallor and cyanosis of the fingers. Also called Raynaud's disease.

aneurysm

A sac formed by the localized dilation of the wall of an artery, a vein, or the heart.

intermittent claudication

A symptom associated with arterial occlusive disease. It refers to pain in the calf of a lower extremity, usually brought on by activity or exercise, and ceases with rest.

The nurse has just completed discharge teaching for a client recently diagnosed with hypertension. Which of the following statements by the client indicate understanding of the Dietary Approaches to Stop Hypertension (DASH) diet? Select all that apply. 1. "I need to eat less red meat and more fresh vegetables." 2. "I'll limit drinking soda to only one at a time as an occasional treat." 3. "I'm going to replace potato chips with fruit during meals and snacking." 4. "I'm really going to miss drinking as much milk as I normally do." 5. "Taking the salt shaker off the table should be enough to reduce my sodium intake."

ANS : 1,2,3 Choosing meats lower in cholesterol (eg, fish, poultry) and alternate protein sources (eg, legumes) instead of red meats (Option 1) Limiting intake of sweets, foods high in sodium (eg, potato chips, frozen meals, canned foods), and sugary beverages to the occasional treat (Options 2 and 3) (Option 4) Limiting milk intake is unnecessary; however, the nurse may need to educate the client about choosing low-fat or skim milk over whole milk. (Option 5) Taking the salt shaker off the table may be a good first step in reducing sodium intake. However, it will not be enough as salt is found in many foods.

The client is scheduled to have a cardiac catheterization. Which findings will cause the nurse to question the safety of the test proceeding? Select all that apply. 1. Elevated C-reactive protein level 2. History of previous reaction to IV contrast 3. Prolonged PR interval on electrocardiogram 4. Serum creatinine of 2.5 mg/dL (221 µmol/L) 5. Took metformin today for type 2 diabetes

ANS : 2,4,5 Allergic reaction: Clients with a previous allergic reaction to IV contrast may require premedication (eg, corticosteroids, antihistamines) or another contrast medium (Option 2). However, clients with renal impairment (eg, serum creatinine >1.3 mg/dL [115 µmol/L]) should not receive IV contrast unless absolutely necessary (Option 4). Metformin is usually discontinued 24-48 hours before exposure and restarted after 48 hours, when stable renal function is confirmed (Option 5). (Option 1) C-reactive protein, produced during acute inflammation, may reflect an elevated risk for coronary artery disease. However, it does not indicate an acute event and is not a safety concern for this procedure. (Option 3) First-degree atrioventricular block (ie, PR interval >0.20 second) may precede more serious conditions. However, clients are usually asymptomatic and do not require treatment except for stopping the causative medication (eg, beta blockers, digoxin). This would not prevent the test from proceeding.

The nurse is caring for a client who, 30 minutes ago, underwent an ablation procedure for supraventricular tachycardia in the cardiac catheterization laboratory. The client has a dressing over the femoral insertion site with a small amount of oozing blood. Which action by the nurse causes the charge nurse to intervene? 1. Applies pressure above the femoral insertion site 2. Assesses bilateral pedal pulses frequently 3. Assists client to sit on the side of the bed to use the urinal 4. Reports client chest pain of 2 on a scale of 0-10 to health care provider

ANS : 3 After cardiac catheterization, clients must remain supine with the head of the bed at ≤30 degrees and the affected extremity straight to prevent bleeding from the catheter insertion site. The charge nurse should intervene if the nurse is assisting the client to sit on the side of the bed to use the urinal (Option 3). (Option 1) A small amount of bleeding can be expected after the catheter is removed. It is appropriate to apply pressure above the insertion site to control bleeding. The nurse should continue to closely monitor the site for further bleeding. (Option 2) It is important to verify adequate perfusion to the affected limb by frequently palpating the pedal pulses. Bilateral pulses should be palpated for comparison. (Option 4) Chest pain after ablation may be due to cardiac muscle damage but could also be caused by cardiac ischemia. This should be reported immediately to the health care provider.

A cardiac catheterization was performed on a client 2 hours ago. The catheter was inserted into the left femoral artery. What signs of potential complications should the nurse report immediately to the health care provider (HCP)? Select all that apply. 1. Bleeding at the catheterization site 2. Client lying down and quietly watching television 3. Client taking only sips of fluids 4. Left foot remarkably cooler than right foot 5. Urine output of 100 mL since the procedure

ANS ; 1,4 Arterial bleeds can lead to hypovolemic shock and death if not treated immediately. Reduced warmth in the lower extremity of the insertion site is a sign of decreased perfusion (lack of oxygenated blood flow) to the extremity and can result in tissue necrosis of the affected area. (Option 2) The client may lie flat for several hours and is encouraged to engage in quiet activities for 24 hours after the procedure to prevent dislodging the clot at the insertion site. (Option 3) Although clients are encouraged to drink fluids to flush dyes out of their system and prevent dehydration, decreased fluid intake would not warrant notifying the HCP. (Option 5) Urine output in this client is above 30 mL/hr and considered to be within the normal range.

A client presents to the clinic with chest pain. For which test should the nurse prepare the client first? 1. Electrocardiogram (ECG) 2. Echocardiogram 3. Tile table test 4. Angiography

ANs: 1 Rationales Option 1: An ECG should be performed first because it can quickly show abnormalities related to conduction, rate, rhythm, heart chamber enlargement, myocardial ischemia, MI, and electrolyte imbalances. Option 2: This test may be performed, but an ECG should be performed first. Option 3: This tests for causes of syncope. Option 4: This test may be performed, but an ECG should be performed first. Test Taking Tip: The least invasive, quickest test is typically performed first.

Rhythm: irregularly irregular Heart rate: atrial rate not measurable; ventricular rate under 100 bpm is controlled response; greater than 100 bpm is rapid ventricular response P waves: no identifiable P waves PRI: none can be measured because no P waves are seen QRS interval: less than or equal to 0.10 seconds

ATRIAL FIBRILLATION RULES.

A client tells the nurse they have atherosclerosis and asks what this means. Which response by the nurse is most appropriate? 1. "This means you have an aneurism that may soon rupture." 2. "You have a rare form of cancer that occurs in the heart." 3. "It is a form of plaque in the arteries that can lead to coronary artery disease." 4. "It is the loss of elasticity of the arterial walls that occurs with age."

Ans #3: Rationales Option 1: This does not describe an aneurysm. Option 2: This does not describe cancer of the heart. Option 3: This describes atherosclerosis. Option 4: This describes arteriosclerosis.

A client is being discharged with Holter monitoring for 48 hours. Which client statement indicates teaching is effective? 1. "I will perform light activities while being monitored." 2. "I will push the event button when I start having symptoms." 3. "The results of the monitoring will determine if I have heart failure." 4. "A shock will be sent to my heart when I have slow heartbeat."

Ans#2 Rationales Option 1: The client should be instructed to continue normal daily activities while wearing the Holter monitor. Option 2: The event button should be pressed at the onset of symptoms, and the client should document the symptoms and the activity that was being performed. Option 3: The results of the monitoring can detect myocardial ischemia or dysrhythmia but is not effective at diagnosing heart failure. Option 4: A defibrillator, not a Holter monitor, delivers a shock to the heart if a shockable dysrhythmia occurs.

The nurse is caring for group of clients. Which client is at highest risk for developing deep vein thrombosis (DVT)? 1. A truck driver 2. A coach 3. A nurse 4. A waitress

Ans: #1 Rationales Option 1: A truck driver sits for long periods of time, so they are at high risk for developing a DVT. Option 2: A coach has a job requiring standing, but they also walk frequently. Option 3: A nurse has a job requiring standing for long periods, but they also walk quite often. Option 4: A waitress has a job requiring standing for long periods, but they also walk a lot, preventing stasis.

The nurse is caring for a group of clients. Which client does the nurse identify as a high risk for developing dilated cardiomyopathy? 1. A client with decreased iron levels 2. A client who has hypotension 3. A client who uses marijuana 4. A client who recently received chemotherapy

Ans: #2 Rationales Option 1: Elevated iron levels place a client at risk for dilated cardiomyopathy. Option 2: Hypertension places a client at risk for dilated cardiomyopathy. Option 3: Cocaine use places a client at risk for dilated cardiomyopathy. Option 4: A client who received chemotherapy is at high risk for developing dilated cardiomyopathy.

The nurse provides postprocedural care for a client who underwent a transesophageal echocardiogram (TEE). Which action should the nurse take? 1. Measure cardiac enzymes 2. Determine whether gag reflex is intact 3. Assess the groin for hemorrhage 4. Obtain orthostatic blood pressures

Ans: #2 Rationales Option 1: There is no indication to obtain cardiac enzymes after a TEE. Option 2: The client's throat is anesthetized before the procedure, so it is priority to assess the client's gag reflex (by touching the back of the throat with a cotton tip swab) after a TEE. If the gag reflex is not intact and the client is given food or fluids, aspiration could occur. Option 3: There is no indication to assess the groin for hemorrhage after a TEE because the probe enters the esophagus, not the femoral artery. Option 4: There is no indication to obtain orthostatic blood pressures after a TEE.

The nurse is reinforcing discharge teaching for a client with an implantable cardioverter defibrillator (ICD). Which client statement indicates that teaching is effective? 1. "The ICD will deliver only one shock most of the time." 2. "The ICD will deliver a shock if my heart rate drops below normal." 3. "It will feel like a kick in the chest when the ICD delivers a shock." 4. "I am glad that the battery will not need replacing."

Ans: #3 Rationales Option 1: Multiple shocks may be necessary to terminate the life-threatening rhythm. Option 2: An electrical shock is delivered when a life-threatening rhythm, such as ventricular fibrillation, occurs. Some ICDs have a combined pacemaker that will generate an electrical impulse and increase the heart rate if it drops too low. Option 3: Many clients experience anxiety and emotional stress because it does feel like a kick in the chest when the ICD delivers an electrical shock. Option 4: The battery will need replacing, and battery life depends on usage.

The nurse is caring for a client receiving heparin sodium who begins bleeding. Which medication is an antidote to heparin? 1. Vitamin K 2. Digoxin immune fab 3. Protamine sulfate 4. Narcan

Ans: #3 Rationales Option 1: Vitamin K is the antidote for coumadin. Option 2: Digoxin immune fab is the antidote for digoxin. Option 3: Protamine sulfate is the antidote for heparin. Option 4: Narcan reverses the effects of opioids.

The nurse is caring for a client with exacerbation of asthma and hypertension. Which medication order should the nurse question? 1. Metoprolol 2. Hydrochlorothiazide 3. Losartan 4. Clonidine

Ans: 1 Option 1: Beta blockers can cause bronchospasm; a client with asthma exacerbation is already experiencing bronchospasm. Option 2: This medication is not contraindicated in this client. Option 3: This medication does not cause bronchospasm. Option 4: This medication is not contraindicated in this client. Test Taking Tip: A client with exacerbation of asthma is likely experiencing bronchospasm.

The nurse is caring for a client with aortic regurgitation. Which of the following clinical manifestations should the nurse expect to find? 1. Forceful heartbeat when lying down 2. A systolic murmur 3. Elevated diastolic blood pressure 4. Thready pulse

Ans: 1 Rationales Option 1: A forceful heartbeat more pronounced when lying down is noted. Option 2: Diastolic murmur is heard. Option 3: Diastolic pressure decreases to compensate for an increase in systolic pressure. Option 4: The pulse is forceful, then quickly collapses (known as Corrigan pulse).

The nurse receives laboratory results on multiple clients. Which laboratory value should be immediately reported to the health-care provider? 1. Magnesium level 1.1 mg/dL 2. Magnesium level 2.6 mg/dL 3. Potassium level 3.7 mEq/L 4. Potassium level 5.0 mEq/L

Ans: 1 Rationales Option 1: A normal magnesium level is 1.6 to 2.6 mg/dL. Hypomagnesemia can cause cardiac arrhythmias, hypertension, and tachycardia. This laboratory value should be immediately reported. Option 2: This client's magnesium level is within normal range. Option 3: The normal potassium level is 3.5 to 5.0 mEq/L. Option 4: The normal potassium level is 3.5 to 5.0 mEq/L.

The nurse is assessing the resting heart rate of a marathon runner. Which heart rate is an expected finding in this client? 1. 50 beats/minute 2. 70 beats/minute 3. 90 beats/minute 4. 110 beats/minute

Ans: 1 Rationales Option 1: An athletic individual typically has a slow heart rate because a well-conditioned heart pumps efficiently. Option 2: This is a normal heart rate, but in a marathon runner, the heart rate is typically 50 because the heart is well-conditioned and pumps efficiently. Option 3: This is a normal heart rate, but a marathon runner would not have a heart rate this high. Option 4: This is a normal heart rate, but a marathon runner would not have a heart rate this high. Test Taking Tip: A well-conditioned heart pumps effectively.

A client is scheduled to be placed on a cardiopulmonary bypass pump (CBP). Which of the following causes the nurse to intervene immediately? 1. Warfarin is administered for anticoagulation 2. Partial thromboplastin time (PTT) is five to six times greater than normal 3. The CBP is primed with lactated Ringer's solution 4. Protamine sulfate is scheduled for administration before the client comes off CBP

Ans: 1 Rationales Option 1: Heparin, not warfarin, is administered before the client goes on the CBP. Option 2: The PTT should be five to six times greater than normal. PTT is used to measure the effectiveness of heparin. Option 3: The CBP is primed with lactated Ringer's solution to minimize the risk of air embolism. Option 4: Protamine sulfate, the antidote for heparin, is administered immediately before the client comes off the CBP. Test Taking Tip: An immediate intervention is required when the client's safety is at risk. The nurse should identify and eliminate answers that do not pose a risk to the client.

Which of the following is the most common cause of left-sided heart failure? 1. Hypertension 2. Kidney failure 3. Mitral stenosis 4. Aortic regurgitation

Ans: 1 Rationales Option 1: High blood pressure is the number-one cause of left-sided heart failure. Option 2: Kidney failure does not cause heart failure directly. Option 3: Mitral stenosis is not the most common cause of left-sided heart failure. Option 4: Aortic regurgitation is not the most common cause of left-sided heart failure.

The nurse is reinforcing discharge teaching for a client being prescribed metoprolol succinate. Which client statement indicates teaching is effective? 1. "I will move slowly when I get out of bed each morning." 2. "I will have to stop the medication if I develop glaucoma." 3. "The medication will affect my blood pressure but not my heart rate." 4. "I may bruise and bleed easily while taking this medication."

Ans: 1 Rationales Option 1: Orthostatic hypotension can occur while taking metoprolol succinate, so the client should change positions slowly. Option 2: Glaucoma is not a contraindication for metoprolol succinate. Option 3: Metoprolol succinate lowers blood pressure and heart rate. Option 4: Metoprolol succinate is a beta blocker and, unlike an anticoagulant, will not cause the client to bruise or bleed easily.

A client returns to the unit immediately after a cardiac catheterization in which the femoral artery was used. Which action should the nurse take first? 1. Palpate peripheral pulses 2. Encourage PO fluids 3. Assist the client with ambulation 4. Encourage the client to lie still and not bend the knees

Ans: 1 Rationales Option 1: Peripheral pulses should be palpated and compared for strength and equality. A decrease in pulse strength indicates impaired circulation. Option 2: The client is not NPO after the procedure and should be encouraged to eat and drink fluids to help eliminate the dye from the body. Option 3: To prevent bleeding, the client is placed on bed rest immediately following a cardiac catheterization. Option 4: Because the client's femoral artery was used for the cardiac catheterization, the client should be on bed rest and instructed to not move or flex the leg for several hours to prevent bleeding. Test Taking Tip: Client assessment is the priority when the client is returned to the unit from a procedure.

The nurse is caring for a client with pulmonary edema. In which position should the nurse place the client? 1. Fowler's 2. Left side-lying 3. Prone 4. Supine

Ans: 1 Rationales Option 1: Semi-Fowler's or high Fowler's will reduce venous return. Option 2: Left side-lying position will not decrease venous return. Option 3: The client should not be placed in the prone position; it will increase venous return. Option 4: The client should not be positioned flat in the bed; it will increase venous return. Test Taking Tip: The goal is to position the client to reduce venous return and allow the lungs to expand easily.

Which of the following is the most common cause of right-sided heart failure? 1. Left-sided heart failure 2. Hypertension 3. Infective endocarditis (IE) 4. Aortic regurgitation

Ans: 1 Rationales Option 1: The most common cause of right-sided heart failure is left-sided heart failure. Option 2: Hypertension is not the most common cause of right-sided heart failure. Option 3: IE is not the most common cause of right-sided heart failure. Option 4: Aortic regurgitation is not the most common cause of right-sided heart failure.

The nurse is teaching a postoperative client how to apply antiembolism stockings. Which statement made by the client indicates a need for further teaching? 1. "The top of the stocking need to be 2 inches over the kneecap." 2. "The stockings should not roll down." 3. "I should turn the stocking inside out to put them on." 4. "I can use a device to help me get these on."

Ans: 1 Rationales Option 1: The stockings should be worn 1 to 2 inches below the kneecap. Option 2: This statement is correct; stockings will cause stasis if rolled down. Option 3: This statement is correct; the stocking should be turned inside out for ease in applying. Option 4: This statement is correct; there are devices used to aid individuals in applying stockings. Test Taking Tip: Antiembolism stockings are used to prevent stasis.

The nurse is teaching a client about prevention of varicose veins. Which statement made by the client indicates the need for further teaching? 1. "I need to avoid compression stockings." 2. "I should exercise more often." 3. "I should elevate my legs when I can." 4. "I need to lose some weight."

Ans: 1 Rationales Option 1: This statement requires further teaching; compression stockings prevent varicose veins. Option 2: This statement is accurate. Option 3: This statement is accurate. Option 4: This statement is accurate.

The nurse is assessing a client with a suspected deep vein thrombosis (DVT) of the left leg. Which clinical manifestations can the nurse expect to find? Select all that apply. 1. Edema 2. Cool skin 3. Pain 4. Calf tenderness 5. Venous distention

Ans: 1, 3, 4, 5 Rationales Option 1: Edema is common in clients with a DVT. Option 2: Warm skin is found in clients with DVT. Option 3: Pain is typical in clients with a DVT. Option 4: Calf tenderness is noted in clients with a DVT. Option 5: Venous distention occurs with a DVT.

The nurse is providing preoperative teaching to a client about to undergo cardiac surgery. Which of the following statements made by the client indicates a need for further education? 1. "I should avoid coughing and deep breathing so my incision does not reopen." 2. "I will have a urinary catheter after surgery." 3. "I will have a tube to help me breathe, so I cannot talk after surgery." 4. "I should alert the nurse if pain medication is not effective."

Ans: 1 Rationales Option 1: This statement requires further teaching; cough and deep breathing will help prevent postoperative complications such as pneumonia. Option 2: This statement is correct; a urinary catheter will be in place following surgery. Option 3: This statement is correct; the client will be ventilated and unable to talk until the ET tube has been removed. Option 4: This statement is correct; the client should inform the nurse if the pain medication given is not effective in reducing pain

The nurse is providing teaching to a client with strep throat about preventing rheumatic fever. Which statement made by the client indicates an understanding of the teaching? 1. "I will be given penicillin to treat the strep throat." 2. "Since I am 17, I cannot get rheumatic fever." 3. "Strep throat does not require antibiotics." 4. "I will need to get a nasal swab to see if I have strep."

Ans: 1 Rationales Option 1: Using antibiotics to treat streptococcal infections can prevent rheumatic fever. Option 2: Although it is more common in ages 5 to 15, it can occur at any age Option 3: Streptococcal infections should be treated quickly with antibiotics. Option 4: A throat culture is used to diagnose strep throat.

The nurse is assessing a client with aortic stenosis. Which of the following should concern the nurse the most? 1. Wet lung sounds 2. Dizziness 3. Exertional dyspnea 4. Activity intolerance

Ans: 1 Rationales Option 1: Wet lung sounds can be indicative of heart failure or pulmonary edema. Option 2: This is a clinical manifestation of early aortic stenosis. Option 3: This is a clinical manifestation of early aortic stenosis. Option 4: This is a clinical manifestation of early aortic stenosis. Test Taking Tip: Some of the options are typical symptoms of clients with aortic stenosis

The first heart sound heard at the beginning of systole when the tricuspid and mitral valves close is known as which sound? 1. S1 2. S2 3. S3 4. S4

Ans: 1 The question describes the S 1 heart sound. Option 2: The S2 heart sound is heard at the start of diastole when aortic and pulmonic semilunar valves close. Option 3: The S3 heart sound is normal for children and young adults. It sounds like a gallop and is low-pitched and heard early in diastole. In older adults, an S3 may be heard with left-sided heart failure. Option 4: The S4 heart sound is also a low-pitched sound similar to a gallop and heard late in diastole. It occurs with hypertension, CAD, and pulmonary stenosis. Test Taking Tip: S1 and S2 are normal heart sounds.

The nurse is providing teaching to a client who will undergo a percutaneous coronary intervention (PCI). Which statement made by the client indicates an understanding of the teaching? 1. "This procedure will open up my blocked artery." 2. "This is used to dissolve the blood clots blocking the artery." 3. "Medication is released over months to reduce risk of restenosis." 4. "I will need to have the PCI within 6 hours."

Ans: 1 Rationales Option 1: This describes a PCI. Option 2: This describes thrombolytic therapy. Option 3: This describes a coronary artery stent. Option 4: A PCI is initiated within 90 minutes.

The nurse is caring for a client with varicose veins. Which clinical manifestations can the nurse expect to find? Select all that apply. 1. Edema 2. Dull pain 3. Feeling of heaviness 4. Spider veins 5. Cyanotic extremities

Ans: 1, 2, 3, 4 Rationales Option 1: Edema is found in clients with varicose veins. Option 2: Dull pain is a common finding. Option 3: This is common; especially after standing. Option 4: Spider veins are commonly seen. Option 5: This is common with arterial insufficiency.

A client's electrocardiogram (ECG) tracing reveals a regular rhythm with a rate of 130 bpm. For which etiology should the nurse assess? Select all that apply. 1. Hypoxia 2. Shock 3. Dehydration 4. Atropine use 5. Anxiety

Ans: 1, 2, 3, 4, 5 Rationales Option 1: Hypoxia causes tachycardia as a compensatory mechanism. Option 2: Shock causes tachycardia as a compensatory mechanism to maintain cardiac output. Option 3: Dehydration causes tachycardia as a compensatory mechanism to maintain cardiac output. Option 4: Medications such as atropine, epinephrine, and nitrates cause tachycardia. Option 5: Anxiety and fear cause tachycardia.

The nurse is caring for a group of clients. Which client does the nurse identify as high risk for developing infective endocarditis (IE)? Select all that apply. 1. A client with human immunodeficiency virus (HIV) 2. A client who reports using intravenous (IV) drugs 3. A client with gingival gum disease 4. A client with hypothyroidism 5. A client with an artificial heart valve

Ans: 1, 2, 3, 5 Rationales Option 1: Clients with a compromised immune system are at risk for developing infective endocarditis. Option 2: An IV drug user is at risk for IE. Option 3: A client with gingival gum disease is at risk for IE. Option 4: A client with hypothyroidism is not at risk for IE. Option 5: Clients with artificial heart valves are at risk for developing infective endocarditis.

A 75-year-old client is undergoing an exercise stress test. Which symptom experienced by the client should cause the stress test be stopped? Select all that apply. 1. Complaints of chest pain 2. Report of leg pain 3. Heart rate 125 bpm 4. Electrocardiogram (ECG) changes 5. Report of inability to exercise further

Ans: 1, 2, 4, 5 Rationales Option 1: Complaints of chest pain indicate the exercise stress test should be stopped. Option 2: A report of leg pain (intermittent claudication) indicates the stress test should be stopped. Option 3: A heart rate of 125 bpm is not a criterion for the exercise stress test to be stopped. The test should be stopped when the client's peak heart rate is reached (client's age subtracted from 220). This client's peak heart rate is 145 bpm. Option 4: ECG changes indicate the stress test should be stopped. Option 5: A report of inability to exercise further indicates the stress test should be stopped.

The nurse is caring for a client with a deep vein thrombosis (DVT). Which interventions should the nurse implement? Select all that apply. 1. Apply compression stockings. 2. Turn the client every 4 hr. 3. Instruct the client to quit smoking. 4. Encourage ambulation as tolerated. 5. Administer warfarin as ordered.

Ans: 1, 3, 4, 5

A client taking warfarin for atrial fibrillation has a critically high international normalized ratio (INR). Which order should the nurse expect? 1. Vitamin K 2. Rivaroxaban 3. Dabigatran 4. Protamine sulfate

Ans: 1. Rationales Option 1: A critically high INR indicates the client's blood is too anticoagulated, and vitamin K is the reversal agent for warfarin. Option 2: Rivaroxaban is an anticoagulant that would further increase the client's risk for bleeding. Option 3: Dabigatran is an anticoagulant that would further increase the client's risk for bleeding. Option 4: Protamine sulfate is the antidote for heparin.

A client presents to the emergency department with chest pain. Which blood study is most important to monitor to rule out a myocardial infarction (MI)? 1. Complete blood count (CBC) 2. Cardiac troponin 3. Kidney function tests 4. Potassium level

Ans: 2 Rationales Option 1: A baseline CBC may be drawn, but it is not used to rule out an MI. Option 2: A cardiac troponin level is drawn to detect cardiac damage. This is more effective than a CK-MB. Option 3: Kidney function tests are not used to diagnose an MI. Option 4: Potassium level will be checked, but an abnormal potassium level does not reveal cardiac damage. A potassium abnormality will show on an ECG.

The nurse is caring for a group of clients. Which client should the nurse see first? 1. A client with mitral valve prolapse who has a murmur 2. A client who underwent valve replacement surgery 2 hr ago who is reporting pain 3. A client with mitral regurgitation awaiting an echocardiogram 4. A client with aortic stenosis who reports dizziness

Ans: 2 Rationales Option 1: A murmur is a clinical manifestation of mitral valve prolapse. Option 2: This client just had surgery and is complaining of pain. This client should be seen first. Option 3: This client is not unstable and does not need to be seen first. Option 4: Dizziness is a clinical manifestation of early aortic stenosis. [Page reference: 382] Test Taking Tip: The priority scale is unstable, then stable; acute, then chronic.

The nurse is caring for a client with myocarditis. Which of the following is an early clinical manifestation of this condition? 1. Pleural friction rub 2. Pharyngitis 3. Osler nodes 4. Petechiae

Ans: 2 Rationales Option 1: A pleural friction rub is seen in clients with pericarditis. Option 2: Pharyngitis is a clinical manifestation of myocarditis. Option 3: Osler nodes are found in clients with infective endocarditis. Option 4: Petechiae are found in clients with infective endocarditis.

The nurse is caring for a group of clients. Which client is at highest risk for having a myocardial infarction (MI) at a younger age? 1. A male who occasionally drinks alcohol 2. A female who smokes and uses oral contraception 3. A female who occasionally smokes and is stressed 4. A male who does not exercise but follows a low-fat diet

Ans: 2 Rationales Option 1: Alcohol does not place a client at risk for an MI. Option 2: This client is at high risk for an MI; female, smoker, and uses oral contraception are all risk factors. Option 3: Smoking and stress place this client at risk but not as high risk as the client who smokes and uses oral contraception. Option 4: This client is not at high risk for an MI.

The nurse is caring for a client receiving anticoagulant therapy for a deep vein thrombosis (DVT) of the left leg. How will the nurse know that therapy has been effective? 1. The affected extremity is warm to the touch. 2. Measurement of the effected calf has shown a decrease in size. 3. Client reports pain of 6 on 0-10 scale. 4. The nurse notes 3+ edema to the affected leg.

Ans: 2 Rationales Option 1: An extremity warm to the touch indicates a DVT. Option 2: If a calf decreases in size, therapy is effective. Option 3: Pain indicates that therapy is not effective. Option 4: 3+ edema indicates DVT.

A client with a diagnosis of hypertension who has been taking medication reports a dry cough. Which medication can the nurse expect to find in the client's medication history? 1. Amlodipine 2. Lisinopril 3. Clonidine 4. Furosemide

Ans: 2 Rationales Option 1: Calcium channel blockers do not cause dry cough. Option 2: The side effect of an ACE inhibitor is dry cough. Option 3: Central-acting alpha2 agonists do not cause dry cough. Option 4: Loop diuretics do not cause dry cough. Test Taking Tip: ACE inhibitors, also known as "Prils," can cause a continuous cough.

A client's electrocardiogram (ECG) tracing shows ventricular fibrillation (VF). What action should the nurse make the first priority? 1. Assist with endotracheal intubation 2. Apply external defibrillation 3. Intravenous (IV) lidocaine administration 4. IV amiodarone administration

Ans: 2 Rationales Option 1: Endotracheal intubation to support respiratory function is an appropriate intervention, but external defibrillation is a higher priority. Option 2: Immediate external defibrillation is the best treatment for terminating VF. Survival is reduced each minute that passes without defibrillation. Option 3: Administering IV lidocaine, an antiarrhythmic, is an appropriate intervention, but external defibrillation is a higher priority. Option 4: Administering IV amiodarone, an antiarrhythmic, is an appropriate intervention, but external defibrillation is a higher priority.

Hypertension can lead to which change in the heart? 1. Increase in size of left atrium 2. Increase in size of left ventricle 3. Increase in size of right atrium 4. Increase in size of right ventricle

Ans: 2 Rationales Option 1: Hypertension does not increase the size of the left atrium. Option 2: High blood pressure can cause an increase in size of the left ventricle (hypertrophy). Option 3: Hypertension does not increase the size of the right atrium. Option 4: Hypertension does not increase the size of the right ventricle.

A client's cardiac rhythm is supraventricular tachycardia (SVT), despite drug therapy. Which action should the nurse take? 1. Prepare for immediate defibrillation 2. Prepare for synchronized cardioversion 3. Prepare to administer intravenous (IV) atropine 4. Prepare to administer IV dopamine

Ans: 2 Rationales Option 1: Immediate defibrillation is indicated for ventricular fibrillation and ventricular tachycardia without a pulse. Option 2: Synchronized cardioversion is used for SVT, atrial fibrillation, and atrial flutter that is not responsive to drug therapy. Synchronized cardioversion is also used for ventricular tachycardia with a pulse. The R wave of the QRS complex must be recognized for a shock to be delivered. Option 3: IV atropine is used to treat sinus bradycardia and is contraindicated for SVT. Option 4: An IV dopamine infusion is used to treat sinus bradycardia and is contraindicated for SVT. [Page reference: 449] Test Taking Tip: Drug therapy has been ineffective, so answer choices that suggest administration of additional medications are eliminated

The nurse is preparing to administer propranolol to a client with hypertension. The nurse obtains a blood pressure reading of 76/42. Which action should the nurse take? 1. Administer only half of the medication 2. Hold the medication and notify the health-care provider (HCP) 3. Administer the medication 4. Recheck the blood pressure in 3 hours

Ans: 2 Rationales Option 1: It is not within the nurse's scope of practice to determine whether giving half of the dose is warranted. Option 2: The nurse should withhold the medication and notify the HCP to determine whether there are any further orders. Option 3: The nurse should not administer the medication with a blood pressure reading that low. Option 4: The nurse should recheck the blood pressure but still needs to withhold the medication and notify the HCP Test Taking Tip: Recall the nurse's scope of practice prior to answering.

The nurse is reviewing lab values for a client taking furosemide. Which of these lab values would concern the nurse the most? 1. Magnesium 1.9 mEq/L 2. Potassium 2.5 mEq/L 3. Sodium 140 mEq/L 4. Calcium 9.4 mg/dL

Ans: 2 Rationales Option 1: Magnesium is within normal range. Option 2: Potassium is very low. Option 3: Sodium is within normal range. Option 4: Calcium is within normal range. Test Taking Tip: Recall the action of furosemide.

The nurse is administering furosemide (Lasix) to a client with heart failure. Which laboratory value would concern the nurse? 1. Sodium 135 mEq/dL 2. Potassium 2.8 mEq/dL 3. Calcium 9.0 mg/dL 4. Magnesium 2.0 mEq/L

Ans: 2 Rationales Option 1: Normal sodium is 135 to 145 mEq/dL. Option 2: Normal potassium is 3.5 to 5.5 mEq/dL. This is low, and Lasix is a potassium-wasting diuretic, which will decrease if the medication is given. Option 3: Normal calcium is 8.6 to 10.2 mg/dL. Option 4: Normal magnesium levels are 1.5 to 2.5 mEq/L.

A client's electrocardiogram (ECG) tracing reveals ventricular fibrillation. Which action should the nurse take first? 1. Insert a large-bore intravenous (IV) and administer epinephrine 2. Apply self-adhesive pads to the client's chest 3. Escort the client's family from the room 4. Provide oxygen with a bag valve mask

Ans: 2 Rationales Option 1: Preparation for immediate defibrillation to terminate the life-threatening rhythm is the top priority. Option 2: Self-adhesive pads should be applied to the client's chest in preparation for immediate defibrillation. Option 3: The client's family requires emotional support, but immediate defibrillation to save the client's life is the top priority. In some facilities, family presence during resuscitation is allowed. Option 4: Preparation for immediate defibrillation to terminate the life-threatening rhythm is the top priority.

Where does systemic circulation begin? 1. Left atrium 2. Left ventricle 3. Right atrium 4. Right ventricle

Ans: 2 Rationales Option 1: Systemic circulation does not begin in the left atrium. Option 2: Systemic circulation begins in the left ventricle; it pumps oxygenated blood into the aorta, the many branches of which eventually give rise to capillaries within the tissues. Option 3: Systemic circulation does not begin in the right atrium. Option 4: Systemic circulation does not begin in the right ventricle.

The nurse is reviewing the prothrombin time (PT) of a client receiving warfarin. The PT is 64 seconds. Which action should the nurse take first? 1. Administer the medication as ordered. 2. Notify the health-care provider (HCP). 3. Withhold the medication. 4. Hold the medication for 1 hr, then administer.

Ans: 2 Rationales Option 1: The blood is too thin; the nurse should notify the HCP. Option 2: The HCP should be notified; the blood is too thin. Option 3: The nurse should first notify the health-care provider. Option 4: The medication should not be given; the blood is too thin. Test Taking Tip: The normal therapeutic range of warfarin is 13.5 to 24 seconds.

The nurse is teaching a client who recently experienced a myocardial infarction (MI) about sexual activity. Which of the following indicates an understanding of the teaching? 1. "I should avoid having sex for at least 6 months." 2. "I can have sex when I can walk up two flights of stairs without symptoms." 3. "I cannot have sex since I have already had an MI. It puts me at risk for another." 4. "There is not a waiting period after having a heart attack and having sex."

Ans: 2 Rationales Option 1: The client does not have to wait 6 months. Option 2: This is accurate; a client can have sex once they can walk up two flights of stairs with no symptoms. Option 3: The client can eventually have sex. Option 4: The client should not have sex immediately.

The nurse is teaching a client about an exercise stress test. Which of the following should the nurse include in the teaching? 1. Avoid eating or drinking for 8 hours before the test. 2. Do not smoke 2 to 4 hours before the test. 3. An electrocardiogram (ECG) will be recorded before and after the test. 4. The test will conclude after reaching a heart rate of 200.

Ans: 2 Rationales Option 1: The client is NPO (nothing by mouth) 2 to 4 hours before the test. Option 2: The client should not smoke 2 to 4 hours before the test. Option 3: An ECG is recorded throughout the test. Option 4: The peak heart rate is the client's age subtracted from 220.

The nurse is caring for a client receiving furosemide. The nurse should provide the following education to the client: 1. Avoid all sources of potassium 2. Take the medication in the morning 3. If edema occurs, double the dose 4. This medication must be taken on an empty stomach

Ans: 2 Rationales Option 1: The client is taking a potassium-wasting diuretic and should consume potassium. Option 2: A diuretic should not be taken at night because it will cause the client to be up all night voiding. Option 3: Furosemide helps to relieve edema but should not be doubled unless instructed by the health-care provider. Option 4: Furosemide should be taken with food or milk. Test Taking Tip: Furosemide is a potassium-wasting diuretic.

The nurse is providing teaching to a client taking bumetanide (Bumex). Which statement made by the client indicates an understanding of the teaching? 1. "I should weigh myself every week while taking Bumex." 2. "I should take this medication early so I don't pee all night." 3. "This medication will cause dry mouth." 4. "Muscle cramps are a normal side effect of this medication."

Ans: 2 Rationales Option 1: The client should weight themselves daily. Option 2: This statement is accurate; the medication should not be taken late in the day because it can cause nocturia. Option 3: Dry mouth can be indicative of electrolyte imbalance. Option 4: Muscle cramps are indicative of hypokalemia, and the health-care provider should be notified.

The nurse is caring for a client who is at risk for orthostatic hypotension. Which action should the nurse take? 1. Assist the client to quickly change positions 2. Use a walking belt when ambulating the client 3. Administer opioid analgesics prior to ambulation 4. Restrict fluids between the hours of 14:00 and 08:00

Ans: 2 Rationales Option 1: The client who is at risk for orthostatic hypotension should never change positions quickly, even with the nurse's assistance. Option 2: For clients at risk for orthostatic hypotension, a walking belt or two-person assist should be used to prevent falls. Option 3: Administering opioid analgesics prior to ambulation increases the risk for orthostatic hypotension. Option 4: Restriction of fluids for an 18-hour period can lead to deficient fluid volume, which is a risk for orthostatic hypotension.

The nurse is applying a nitroglycerin patch (Nitro-Dur) for a client. Which of the following requires correction? 1. The nurse removes the old patch before applying the new one. 2. The nurse applies the patch at the same location as the old patch. 3. The nurse applies the patch to a clean, dry, hairless area. 4. The nurse removes the patch at bedtime

Ans: 2 Rationales Option 1: The old patch should be removed before placing the new one. Option 2: The locations should be rotated. Option 3: The patch should be placed on a clean, dry, and hairless area. Option 4: The patch should be removed at bedtime to prevent tolerance.

The nurse is applying a nitroglycerin patch (Nitro-Dur) for a client. Which of the following requires correction? 1. The nurse removes the old patch before applying the new one. 2. The nurse applies the patch at the same location as the old patch. 3. The nurse applies the patch to a clean, dry, hairless area. 4. The nurse removes the patch at bedtime.

Ans: 2 Rationales Option 1: The old patch should be removed before placing the new one. Option 2: The locations should be rotated. Option 3: The patch should be placed on a clean, dry, and hairless area. Option 4: The patch should be removed at bedtime to prevent tolerance.

Which part of the heart pumps deoxygenated blood toward the lungs? 1. Right atrium 2. Right ventricle 3. Left atrium 4. Left ventricle

Ans: 2 Rationales Option 1: The right atrium does not pump deoxygenated blood toward the lungs. Option 2: Pulmonary circulation begins at the right ventricle, which pumps deoxygenated blood toward the lungs for gas exchange. Option 3: Oxygenated blood returns to the left atrium by way of the pulmonary veins. Option 4: The left ventricle does not pump deoxygenated blood to the lungs.

The nurse is auscultating a client's heart sounds and notes a prolonged, faint swishing sound. The nurse suspects the client may be exhibiting which abnormality? 1. Pericardial friction rub 2. Murmur 3. Atrial fibrillation (A-fib) 4. Atrial flutter

Ans: 2 Rationales Option 1: This is a sound like sandpaper rubbing together and results from inflammation of the pericardium. Option 2: The nurse is hearing a murmur, which presents as a prolonged swishing sound ranging from faint to loud. Option 3: A-fib is related to electrical impulses that have become chaotic and result in an irregular heartbeat. Option 4: The client with atrial flutter will have a very fast heart beat due to an abnormal conduction circuit inside the right atrium.

The nurse is taking a health history of a client with suspected heart failure. Which question assesses respiratory function? 1. "Have you had a decrease in daytime urine output?" 2. "How many flights of stairs can you climb without dyspnea?" 3. "Have you had any weight gain recently?" 4. "Do you experience nausea or vomiting?

Ans: 2 Rationales Option 1: This question assesses urinary function. Option 2: This question assesses respiratory function. Option 3: This question assesses fluid retention . Option 4: This question assesses gastrointestinal (GI) function. Test Taking Tip: Be sure to choose an option that answers what the stem is asking (respiratory function). You can eliminate options that are not respiratory related.

The nurse is caring for a client with angina. Which medication would the nurse expect to administer? 1. Colestipol (Colestid) 2. Nitroglycerin (Nitrostat) 3. Clopidogrel (Plavix) 4. Fenofibrate (TriCor)

Ans: 2 The nurse is caring for a client with angina. Which medication would the nurse expect to administer? 1. Colestipol (Colestid) 2. Nitroglycerin (Nitrostat) 3. Clopidogrel (Plavix) 4. Fenofibrate (TriCor)

The nurse assists with discharge planning of a client with cardiovascular disease. Which teaching should the nurse reinforce? Select all that apply. 1. Encourage 2 to 3 daily servings of fruits and vegetables. 2. Smoking reduces blood flow and can worsen symptoms. 3. The top of elastic stockings should be 1 to 2 inches below the bottom of the kneecap. 4. Avoid open flames when oxygen is in use. 5. Take simvastatin with grapefruit juice.

Ans: 2, 3, 4 Rationales Option 1: At least 5 daily servings of fruits and vegetables are encouraged. Option 2: Smoking causes vasoconstriction, which reduces blood flow and can worsen symptoms. Vasoconstriction lasts up to 1 hour after smoking one cigarette. Option 3: To prevent venous stasis, the top of elastic stockings should be 1 to 2 inches below the bottom of the kneecap. Stockings that are rolled down will cause, not prevent, venous stasis. Option 4: Open flames and smoking are avoided when oxygen is in use. Option 5: Grapefruit juice, when taken with statins such as simvastatin, causes increased drug absorption in the bloodstream.

A client is scheduled for a cardiac catheterization in 1 hour. Which action should the nurse take? Select all that apply. 1. Report a client allergy to dipyridamole 2. Ensure informed consent is obtained 3. Measure orthostatic blood pressures 4. Report a client allergy to shellfish 5. Discontinue all intravenous (IV) fluids

Ans: 2, 4 Rationales Option 1: Dipyridamole is a coronary vasodilator that can be administered during an exercise stress test if the client is physically unable to exercise. Dipyridamole is not administered during a cardiac catheterization. Option 2: Cardiac catheterization is an invasive procedure that carries risks, so an informed consent should be obtained by the health-care provider performing the procedure. The nurse should verify that the informed consent has been obtained. Option 3: There is no indication to measure orthostatic blood pressures before a cardiac catheterization. Option 4: Contrast dye is injected during the cardiac catheterization to visualize the heart chambers and vessels, so the nurse should report an allergy. Option 5: The client is NPO for about 8 hours before the procedure, so IV fluids may be needed before the procedure while the client is NPO.

A client who is asymptomatic has an irregularly irregular rhythm with no identifiable P waves. Which initial order should the nurse expect? Select all that apply. 1. Catheter ablation 2. Anticoagulant therapy 3. Synchronized cardioversion 4. Beta blocker 5. Calcium channel blocker

Ans: 2, 4, 5 Rationales Option 1: Catheter ablation is not an initial treatment for an asymptomatic client with atrial fibrillation. Option 2: An irregularly irregular rhythm with no identifiable P waves describes atrial fibrillation. To reduce thrombi and stroke risk, anticoagulant therapy is prescribed. Option 3: Synchronized cardioversion is immediately used if the client is unstable, but this client is asymptomatic. Option 4: A beta blocker medication may be prescribed to control the ventricular rate. Option 5: A calcium channel blocker medication may be prescribed to control ventricular rate. Test Taking Tip: An initial order is usually one that, as long as the client is stable, is the least invasive.

The nurse reinforces preoperative teaching for a client scheduled for cardiopulmonary bypass. Which client statement indicates teaching is effective? Select all that apply. 1. "I'll be given a blood thinner before surgery." 2. "My heart will be stopped during surgery." 3. "I can talk to my family immediately after surgery." 4. "There will be a small incision in my groin." 5. "I need to cough and take deep breaths after surgery."

Ans: 2, 4, 5 Rationales Option 1: The client will receive heparin before going on the cardiopulmonary bypass pump. Option 2: A cardioplegic solution and iced saline are infused into the aortic root to cause cardiac standstill during the procedure. Option 3: The client may still have an endotracheal tube in place immediately after surgery, so the client's ability to speak will be impaired. Option 4: A sternotomy is performed for direct access to the heart. The health-care provider does not make an incision into the groin. Option 5: Preoperative teaching of coughing and deep breathing reduces postoperative complications, such as pneumonia.

The nurse is caring for a client with an abdominal aortic aneurysm (AAA). Which finding should the nurse report to the health-care provider immediately? 1. A mild headache 2. Bounding pulses 3. Sudden back pain 4. A dry cough

Ans: 3 Rationales Option 1: A mild headache is not indicative of AAA rupture. Option 2: Bounding pulses are not indicative of complications. Option 3: Sudden pack pain should be reported; it can be a sign of a rupture. Option 4: A dry cough is not indicative of complications with an AAA.

The nurse obtains a blood pressure (BP) reading of 134/86. According to the blood pressure categories, this client's blood pressure is in which category? 1. Normal 2. Elevated 3. Stage I Hypertension 4. Stage II Hypertension

Ans: 3 Rationales Option 1: A normal blood pressure is less than 120 systolic and less than 80 diastolic. Option 2: An elevated blood pressure is a systolic between 120 and 129 and a diastolic less than 80. Option 3: The systolic BP is between 130 and 139, while the diastolic is between 80 and 89. Option 4: The systolic BP is 140 or higher, and the diastolic is 90 or higher.

The nurse is caring for a client with pericarditis who develops cardiac tamponade. For which treatment should the nurse prepare the client? 1. Valvuloplasty 2. Intravenous (IV) antibiotics 3. Pericardiocentesis 4. Myectomy

Ans: 3 Rationales Option 1: A valvuloplasty involves a percutaneous balloon (a balloon that dilates the stenosed heart valve). Option 2: IV antibiotics are not used to treat a cardiac tamponade; it requires invasive treatment. Option 3: A pericardiocentesis is the process of puncturing the pericardium with a needle and draining excess fluid from the pericardial sac. Option 4: A myectomy is a procedure to remove part of the hypertrophied ventricular septum.

The nurse is reviewing orders for a client with angina and a history of asthma. Which order should the nurse question? 1. Amlodipine (Norvasc) 2. Captopril (Capoten) 3. Propranolol (Inderal) 4. Isosorbide dinitrate (Isordil)

Ans: 3 Rationales Option 1: Clients with asthma can take Norvasc. Option 2: Clients with asthma can take Capoten. Option 3: This is a nonselective beta blocker and is contraindicated in clients with asthma. Option 4: This medication is not contraindicated with asthma. Test Taking Tip: Nonselective beta blockers are contraindicated in clients with asthma or chronic obstructive pulmonary disease (COPD).

The nurse is caring for a client with aortic regurgitation. Which of the following interventions should the nurse implement? 1. Encourage the client to complete all activities at once. 2. Place the client on bedrest. 3. Schedule periods of rest between activities. 4. Administer oxygen while the client completes activities.

Ans: 3 Rationales Option 1: Completing the activities all at once may cause further dyspnea and fatigue. Option 2: The client may not require bedrest; scheduling periods of rest may allow them to complete ADLs. Option 3: Scheduling periods of rest between activities may prevent dyspnea and fatigue. Option 4: The nurse can administer oxygen only with an order. Scheduling activities with periods of rest may be helpful.

The nurse is caring for a client with pericarditis. Which of the following is a clinical manifestation the nurse can expect to find? 1. Hemoptysis 2. Petechiae 3. Pericardial friction rub 4. Hypertension

Ans: 3 Rationales Option 1: Hemoptysis is seen in clients with mitral stenosis. Option 2: Petechiae is seen in clients with infective endocarditis. Option 3: A pericardial friction rub is a clinical manifestation in clients with pericarditis. Option 4: Hypertension is seen with aortic regurgitation.

The nurse is reviewing a client's laboratory results. Because the client is taking digoxin, which electrolyte imbalance most concerns the nurse? 1. Hypermagnesemia 2. Hypocalcemia 3. Hypokalemia 4. Hypernatremia

Ans: 3 Rationales Option 1: Hypomagnesemia, not hypermagnesemia, increases the risk for digoxin toxicity. Option 2: Hypercalcemia, not hypocalcemia, increases the risk for digoxin toxicity. Option 3: Hypokalemia increases the risk for digoxin toxicity. Option 4: Sodium imbalances do not directly increase the risk for digoxin toxicity.

The nurse is caring for a client with an arterial occlusion of the left leg. Which intervention should the nurse plan to implement? 1. Apply ice to the extremity. 2. Apply oxygen at 2 L/min via nasal cannula. 3. Administer heparin intravenously as ordered. 4. Encourage ambulation.

Ans: 3 Rationales Option 1: Ice should not be applied to the extremity. Option 2: Oxygen may not be necessary. Option 3: Heparin will be administered to prevent further clotting, followed by warfarin (Coumadin). Option 4: The client should not ambulate; the thrombosis could become an embolism.

Which new class of medications used to treat heart failure (HF) has reduced hospitalizations and deaths from chronic HF? 1. Inotropic agents 2. Beta-adrenergic blockers 3. Angiotensin receptor neprilysin inhibitors (ARNIs) 4. Diuretics

Ans: 3 Rationales Option 1: Inotropic agents are older drugs that strengthen ventricular contraction. Option 2: Beta-adrenergic blockers improve cardiac output, reduce symptoms, reduce disease progression, and reduce sudden death. Option 3: This new medication class of ARNIs has been found to reduce hospitalizations and deaths from HF and reduced ejection fraction. Option 4: Diuretics reduce fluid volume and increase urine output, reducing pulmonary venous pressure.

The nurse is providing teaching to a client who will be discharged taking clonidine. Which statement made by the client indicates an understanding of the teaching? 1. "I should avoid all foods high in potassium." 2. "I will need to weigh myself every day." 3. "I should suck on hard candy if my mouth is dry." 4. "I should wear sunscreen to prevent photosensitivity."

Ans: 3 Rationales Option 1: It is not necessary to avoid sources of potassium while taking this medication. Option 2: It is not necessary to monitor weight daily on this medication. Option 3: This medication causes dry mouth; gum or hard candy should be suggested to reduce this side effect. Option 4: This medication does not cause photosensitivity.

The nurse reinforces teaching for a client scheduled for a cardiac catheterization. Which client statement indicates that teaching is effective? 1. "The procedure will take 20 to 30 minutes." 2. "I will receive general anesthesia for the procedure." 3. "My skin will become flushed and feel warm." 4. "I can't drink fluids for 3 hours before the procedure."

Ans: 3 Rationales Option 1: The procedure will take 2 to 3 hours. Option 2: A local anesthetic will be used to numb the catheter insertion site, but the client will remain awake during the procedure. Option 3: Dye is injected to visualize the heart chambers and vessels, and the dye causes a warm, flushing sensation when injected. Option 4: The client will be NPO for about 8 hours before the procedure.

The nurse is caring for a group of clients. Which client should the nurse see first? 1. A client with varicose veins 2. A client with peripheral arterial disease 3. A client with chest pain radiating to the shoulder 4. A client with an abdominal aortic aneurysm (AAA)

Ans: 3 Rationales Option 1: This client does not need to be seen immediately. Option 2: This client does not need to be seen immediately. Option 3: This client should be seen immediately (may be experiencing a myocardial infarction). Option 4: This client should be seen next, but there is nothing to indicate the abdominal aortic aneurysm (AAA) is unstable or rupturing.

The nurse is monitoring a client's pulse and notes there are fewer radial beats than apical beats. Which action by the nurse is best? 1. Document the finding as normal 2. Continue to monitor the pulses more frequently 3. Notify the health-care provider 4. Instruct the client to stand, and recheck

Ans: 3 Rationales Option 1: This is not a normal finding; the client has a pulse deficit. Option 2: The nurse may be instructed to monitor the pulses more often, but the health-care provider (HCP) must be notified, Option 3: The client has a pulse deficit, and the HCP should be notified. Option 4: The client has a pulse deficit, and the HCP should be notified. The client should be checked for orthostatic hypotension while sitting, standing, and lying. Test Taking Tip: When the radial pulse and apical pulse are unequal, the client has a pulse deficit.

The nurse is teaching a client about enoxaparin. Which statement made by the client indicates the need for further teaching? 1. "I will take this twice per day until my health-care provider tells me I'm done." 2. "Even though it is rare, I should still look for signs of bleeding." 3. "I need to get the air bubble out of the syringe." 4. "I will inject this medication into my fatty tissue."

Ans: 3 Rationales Option 1: This statement indicates compliance and is accurate. Option 2: This is an accurate statement. Option 3: This statement requires correction; the air bubble remains in the syringe. Option 4: This statement is accurate.

The nurse is providing teaching to a client with an abdominal aortic aneurysm. Which statement made by the client indicates a need for further teaching? 1. "I can ride my bike for a short time each day." 2. "I have been taking the medicine for my blood pressure." 3. "I still work at my moving company business." 4. "It is hard, but I need to quit smoking."

Ans: 3 Rationales Option 1: This statement indicates understanding. Option 2: This statement indicates understanding. Option 3: This statement indicates a need for further teaching; heavy lifting should be avoided. Option 4: This statement indicates understanding

After a pacemaker placement, a client is ready for discharge. Which client statement indicates additional teaching is needed? Select all that apply. 1. "My pacemaker will be periodically checked." 2. "I need to carry a pacemaker identification card." 3. "I can no longer use a cell phone or microwave." 4. "Dizziness and chest pain are normal sensations." 5. "Discharge at the incision site is a sign of healing."

Ans: 3, 4, 5 Rationales Option 1: The health-care provider (HCP) will periodically check the pacemaker either in the office or remotely from home. The HCP can reprogram the pacemaker, if needed. Option 2: The client should carry a pacemaker identification card and show it to HCPs and airport security (pacemaker metal may set off alarms). Option 3: Cell phones less than 3 watts, microwaves, and most common household devices are safe to use. Option 4: Sensations such as dizziness, chest pain, fainting, irregular heartbeats, and palpitations should be reported to the HCP because these symptoms indicate possible pacemaker failure. Option 5: Fever and discharge, redness, swelling, warmth, and pain at the incision site are signs of infection that should be reported to the HCP.

Which client is at highest risk for developing heart failure? 1. A Caucasian woman 2. A Hispanic male 3. An Asian male 4. An African American woman

Ans: 4 Rationales Option 1: A Caucasian woman is not as high of a risk for heart failure as an African American woman. Option 2: A Hispanic male is not as high of a risk for heart failure as an African American woman. Option 3: An Asian male is not as high of a risk for heart failure as an African American woman. Option 4: An African American woman has the highest incidence of heart failure.

The nurse is caring for a client with an abdominal aortic aneurysm (AAA). Which of the following should the nurse report to the health-care provider? 1. Anxiety 2. Nausea 3. Feeling of fullness 4. Sudden abdominal pain

Ans: 4 Rationales Option 1: A client with an AAA is likely to have anxiety that the aneurysm may rupture. Option 2: Nausea is common in clients with an AAA. Option 3: Feeling full is a common symptom of clients with an AAA. Option 4: Sudden back, flank, or abdominal pain should be reported immediately; it could be indicative of a ruptured aneurysm.

The nurse reviews a client's electrocardiogram (ECG) tracing.Which action should the nurse take? 1. Administer intravenous (IV) magnesium 2. Prepare for synchronized cardioversion 3. Administer IV 0.9% sodium chloride 4. Obtain the client's potassium level

Ans: 4 Rationales Option 1: Hypomagnesemia does not cause a U wave, nor does IV magnesium treat the possible underlying cause. Option 2: Synchronized cardioversion is not indicated in this situation. Option 3: IV 0.9% sodium chloride does not treat the possible underlying cause. Option 4: Hypokalemia can cause a U wave on an ECG tracing, so the client's potassium level should be checked.

A client has no palpable pulse and no waveform activity on an electrocardiogram (ECG) tracing. Which action should the nurse make the top priority? 1. Administer intravenous (IV) epinephrine 2. Assist with endotracheal intubation 3. Prepare for immediate defibrillation 4. Start chest compressions

Ans: 4 Rationales Option 1: IV epinephrine is part of advanced cardiac life support (ACLS) protocol but starting chest compressions should be the nurse's priority. Option 2: Endotracheal intubation is part of ACLS protocol, and the nurse's assistance may be required, but starting chest compressions should be the nurse's priority. Option 3: Immediate defibrillation is indicated for ventricular fibrillation or pulseless ventricular tachycardia. Option 4: Starting chest compressions immediately increases tissue perfusion and the likelihood of client survival.

The nurse is caring for a client with Raynaud disease. Which of the following clinical manifestations of the affected area can the nurse expect to find? 1. Numbness when warm 2. Red skin when cold 3. Tingling when cold 4. Pain when warm

Ans: 4 Rationales Option 1: Numbness occurs when the area is cold. Option 2: Red skin occurs when the area is warm. Option 3: Tingling occurs when warming. Option 4: Pain occurs when warming.

The nurse is reviewing laboratory values for a client with heart failure. Which of the following should the nurse expect to find? 1. Elevated red blood cells (RBCs) 2. Decreased blood urea nitrogen (BUN) 3. Decreased serum cystatin C 4. Elevated brain natriuretic peptide (BNP)

Ans: 4 Rationales Option 1: RBCs are decreased. Option 2: BUN is elevated. Option 3: Serum cystatin C is increased. Option 4: BNP is elevated.

A client started having chest pain 2 hours ago. After the health-care provider explains to the client that her cardiac troponin level is normal," the client states, "I'm so happy that I didn't have a heart attack." Which response by the nurse is the priority? 1. "Let's discuss the heart benefits of stopping smoking." 2. "You may need to follow-up with a cardiologist after discharge." 3. "A healthy, balanced diet reduces your risk for coronary artery disease." 4. "The troponin level may be normal right now even if you have had a heart attack.

Ans: 4 Rationales Option 1: Smoking cessation decreases the risk of a heart attack, but this response does not provide necessary information to the client right now regarding a possible heart attack. Option 2: Follow-up with a cardiologist may be needed, but this response does not provide necessary information to the client right now regarding a possible heart attack. Option 3: A healthy, balanced diet reduces the risk of coronary artery disease, but this response does not provide necessary information to the client right now regarding a possible heart attack. Option 4: The troponin level elevates within 4 to 6 hours of cardiac muscle damage. The client's chest pain started 2 hours ago, so the troponin level may not be elevated yet even if the client has had a heart attack. The health-care provider will likely order serial troponin levels.

A client started having chest pain 2 hours ago. After the health-care provider explains to the client that her cardiac troponin level is normal," the client states, "I'm so happy that I didn't have a heart attack." Which response by the nurse is the priority? 1. "Let's discuss the heart benefits of stopping smoking." 2. "You may need to follow-up with a cardiologist after discharge." 3. "A healthy, balanced diet reduces your risk for coronary artery disease." 4. "The troponin level may be normal right now even if you have had a heart attack."

Ans: 4 Rationales Option 1: Smoking cessation decreases the risk of a heart attack, but this response does not provide necessary information to the client right now regarding a possible heart attack. Option 2: Follow-up with a cardiologist may be needed, but this response does not provide necessary information to the client right now regarding a possible heart attack. Option 3: A healthy, balanced diet reduces the risk of coronary artery disease, but this response does not provide necessary information to the client right now regarding a possible heart attack. Option 4: The troponin level elevates within 4 to 6 hours of cardiac muscle damage. The client's chest pain started 2 hours ago, so the troponin level may not be elevated yet even if the client has had a heart attack. The health-care provider will likely order serial troponin levels.

A client's electrocardiogram (ECG) tracing reveals no relationship between P waves and QRS complexes. The client is confused, and blood pressure is 60/30 mmHg. Which order should the nurse expect? 1. Synchronized cardioversion 2. Intravenous (IV) atropine 3. IV epinephrine 4. Transcutaneous pacing

Ans: 4 Rationales Option 1: Synchronized cardioversion is not indicated in this situation. Option 2: IV atropine is not indicated in this situation. Option 3: IV epinephrine is not indicated in this situation. Option 4: No relationship between P waves and QRS complexes describes complete heart block, which is a medical emergency. Immediate transcutaneous pacing is required because the client has signs of poor perfusion

An older adult client reports coughing and dyspnea. During assessment, the nurse auscultates S1, S2, and S3 heart sounds and crackles in the lung fields. What is the nurse's priority action? 1. Ask whether the client has an allergy to antibiotics 2. Draw blood to measure cardiac biomarkers 3. Ask whether the client has ever had pneumonia 4. Prepare the client for a chest radiograph

Ans: 4 Rationales Option 1: The client has signs of fluid volume overload and heart failure, not pneumonia. Option 2: The health-care provider may order cardiac biomarkers to rule out a myocardial infarction, but the client's symptoms are more indicative of fluid volume overload and heart failure. Option 3: The client has signs of fluid volume overload and heart failure, not pneumonia. Option 4: The client has signs of fluid volume overload and heart failure (S3 heart sound, crackles in lung fields, cough, dyspnea), so a chest radiograph (chest x-ray) will be ordered to verify the suspicion.

A client returns to the clinic for a follow-up after being treated for hypertension for 4 months. Which assessment data indicates that the medication has been effective? 1. Weight loss of 2 pounds 2. Client reports riding a bicycle for 30 minutes daily 3. Client reports eating food from the DASH diet guidelines 4. Blood pressure is 116/70

Ans: 4 Rationales Option 1: The client is eating healthier and riding a bicycle, contributing to weight loss. Option 2: Riding a bicycle indicates the client is not leading a sedentary lifestyle. Option 3: Choosing food items from the DASH guidelines contributes to weight loss. Option 4: Blood pressure within normal range indicates that the medication has been effective. Test Taking Tip: Recall the goal of an antihypertensive medication.

The nurse is caring for a client with aortic stenosis. Which of the following interventions should the nurse implement? 1. Administer oxygen at 4 L via nasal cannula. 2. Elevate the head of bed 30 degrees. 3. Monitor for severe hypertension. 4. Encourage the client to use the call light when getting up.

Ans: 4 Rationales Option 1: The nurse cannot administer oxygen without an order. Option 2: The head of bed should be raised to 45 degrees to reduce venous return to the heart and improve chest expansion, which increases the amount of oxygen coming into the lungs. Option 3: Hypertension is typically not associated with aortic stenosis. Option 4: Dizziness and syncope are symptoms of aortic stenosis; the nurse should encourage the client to call when getting up.

The nurse is caring for a group of clients. Which client is at highest risk for developing Buerger disease? 1. A client with hepatitis who drinks often 2. A client with anorexia who occasionally smokes 3. A client with peptic ulcer disease who drinks socially 4. A client with lupus who is a heavy smoker

Ans: 4 Rationales Option 1: This client is not at high risk for Buerger disease. Option 2: This client is slightly at risk (smoking). Option 3: This client is not at high risk for Buerger disease. Option 4: This client is at highest risk for Buerger disease (autoimmune disease and heavy smoker). Test Taking Tip: All use of tobacco must be avoided.

The nurse is caring for a client with mitral regurgitation. The client asks the nurse what this means. Which response by the nurse is best? 1. "One or more mitral valve flaps bulge backward into the left atrium during systole. If the bulging flaps do not fit together, blood will leak backward into the left atrium." 2. "The blood flow from the left ventricle into the aorta is obstructed through the stenosed aortic valve. The opening of the aortic valve can be narrowed from thickening of the valve's flaps." 3. "The mitral valve flaps thicken, and the chordae tendineae shorten. This causes the mitral valve opening to narrow." 4. "Your mitral valve leaflets do not close all the way. This allows backflow of blood into the left atrium every time the left ventricle contracts."

Ans: 4 Rationales Option 1: This describes mitral valve prolapse. Option 2: This describes aortic stenosis. Option 3: This describes mitral stenosis. Option 4: This describes mitral regurgitation.

The nurse is providing teaching regarding healthy food choices to a client with hypertension. Which food choice indicates that the client understood the teaching? 1. One medium bean burrito with small French fries 2. One large deli ham sandwich with two slices of cheese 3. ½ cup of canned vegetables with 1 cup of chicken noodle soup 4. 1 small apple with 1 cup of raw leafy vegetables and 1 ½ ounce of cheese

Ans: 4 Rationales Option 1: This food choice is high in fat and sodium. Option 2: This food choice is high in sodium. Option 3: This food choice is high in sodium. The client should choose fresh vegetables instead of canned to reduce sodium. Option 4: This food choice follows the DASH guidelines. Test Taking Tip: Canned foods typically are high in sodium

The nurse is providing teaching to a client about chest pain. Which of the following statements made by the client would be of most concern to the nurse? 1. "If I have chest pain I call 911 right away." 2. "I chew an adult aspirin as soon as I begin to have chest pain." 3. "I have a family history of heart attacks, so I am at risk." 4. "I have chest pain a lot, but it is from indigestion."

Ans: 4 Rationales Option 1: This is an accurate statement. Option 2: This statement is correct. Option 3: This is a true statement. Option 4: If a client continues to have chest pain, it should be investigated, not attributed immediately to indigestion.

The nurse is caring for a client taking digoxin. Which medication in the client's orders should the nurse question? 1. Furosemide 2. Prazosin 3. Quinapril 4. Amlodipine

Ans: 4 Rationales Option 1: This medication does not interact with digoxin. Option 2: This medication does not interact with digoxin. Option 3: This medication does not interact with digoxin. Option 4: Amlodipine may increase blood levels of digoxin. Test Taking Tip: Calcium channel blockers may increase blood levels of digoxin.

The nurse is preparing to measure orthostatic blood pressures on a client who fell. In which order should the nurse perform the following actions? 1 Measure the blood pressure (BP) and HR in the supine position 2 Report abnormal findings to the health-care provider 3 Measure the BP and HR in the sitting position 4 Explain the procedure to the client

Ans: 4, 1, 3, 2 The nurse should first explain the procedure to the client and determine whether the client can safely change positions and stand (if the client cannot, then orthostatic blood pressure measurements are not obtained). The nurse should measure the client's BP and HR in the supine position and then in the sitting position. Lastly, abnormal findings are reported to the health-care provider.

The nurse is providing teaching for a client with heart failure about decreasing their oxygen consumption. Which statement made by the client indicates a need for further teaching? 1. "I should exercise at least 30 minutes per day." 2. "If I put objects at waist level, my heart rate won't rise too much." 3. "I should take stool softeners so I don't strain when having a bowel movement." 4. "I need to take a bath, then rest for a little while, then brush my teeth.'

Ans: A Rationales Option 1: The client should not exercise; tachycardia increases oxygen consumption by the heart and should be avoided. Option 2: The client should place objects at waist level to avoid reaching. Option 3: Avoiding straining when defecating will help reduce oxygen consumption. Option 4: Planning activities with periods of rest helps reduce oxygen consumption.

A client with a regular heart rhythm of 155 bpm reports dyspnea and angina. Which prescribed medications will the nurse expect to administer? 1. Epinephrine 2. Atropine 3. Dopamine 4. Adenosine

Ans: Adenosine Rationales Option 1: Epinephrine may be given for symptomatic bradycardia. Option 2: Atropine may be given for symptomatic bradycardia. Option 3: Dopamine may be given for symptomatic bradycardia. Option 4: Adenosine, beta blockers, or calcium channel blockers may be given for symptomatic tachycardia. The nurse should try to identify the cause of the tachycardia. Test Taking Tip: When the nurse expects to receive a medication order, the medication will treat the client's current condition. The client's heart rate is fast, so the nurse should identify the therapeutic effects of each medication that is listed.

The nurse is providing care for a patient with high cholesterol and triglyceride values. In teaching the patient about therapeutic lifestyle changes such as diet and exercise, the nurse realizes that the desired goal for cholesterol levels is which of the following? A) High HDL values and high triglyceride values B) Absence of detectable total cholesterol levels C) Elevated blood lipids, fasting glucose less than 100 D) Low LDL values and high HDL values

Ans: D Feedback: The desired goal for cholesterol readings is for a patient to have low LDL and high HDL values. LDL exerts a harmful effect on the coronary vasculature because the small LDL particles can be easily transported into the vessel lining. In contrast, HDL promotes the use of total cholesterol by transporting LDL to the liver, where it is excreted. Elevated triglycerides are also a major risk factor for cardiovascular disease. A goal is also to keep triglyceride levels less than 150 mg/dL. All individuals possess detectable levels of total cholesterol.

The nurse is teaching a client with heart failure who just had a myocardial infarction (MI) about nutrition. Which of the following should the nurse include in the teaching? 1. It is important to avoid caffeine. 2. A high-sodium diet should be followed. 3. Foods high in cholesterol will reduce blockage. 4. Fluids should be increased.

Ans: Option 1 Rationales Option 1: Caffeine causes vasoconstriction and should be avoided. Option 2: A low-sodium diet should be followed. Option 3: Foods low in cholesterol will reduce blockage. Option 4: The client has heart failure, so fluids should be restricted, not increased.

A client's cardiac rhythm is supraventricular tachycardia (SVT), despite drug therapy. Which action should the nurse take? 1. Prepare for immediate defibrillation 2. Prepare for synchronized cardioversion 3. Prepare to administer intravenous (IV) atropine 4. Prepare to administer IV dopamine

Ans: Option 2 Rationales Option 1: Immediate defibrillation is indicated for ventricular fibrillation and ventricular tachycardia without a pulse. Option 2: Synchronized cardioversion is used for SVT, atrial fibrillation, and atrial flutter that is not responsive to drug therapy. Synchronized cardioversion is also used for ventricular tachycardia with a pulse. The R wave of the QRS complex must be recognized for a shock to be delivered. Option 3: IV atropine is used to treat sinus bradycardia and is contraindicated for SVT. Option 4: An IV dopamine infusion is used to treat sinus bradycardia and is contraindicated for SVT.

A client scheduled for the next dose of atenolol has a blood pressure (BP) of 94/60 mm Hg. Which action should the nurse take next? 1. Administer the medication, as prescribed 2. Notify the health-care provider 3. Obtain an electrocardiogram (ECG) 4. Administer oxygen at 2L per minute

Ans: Option 2 Rationales Option 1: The BP is lower than the set parameter, so it is unsafe to administer atenolol, a beta blocker medication. Option 2: The nurse should check the BP before giving the medication, and the health-care provider should be notified if the systolic BP is less than 100 mm Hg or the heart rate is less than 50 or 60 bpm. Option 3: The BP is lower than the set parameter, but there is no indication for an ECG at this time. Option 4: The BP is lower than the set parameter, but there is no report of dyspnea, nor is there clear evidence of poor tissue perfusion.

The nurse is caring for a client receiving enalapril (Vasotec). The nurse should expect the client to report which side effect? 1. Indigestion 2. Low back pain 3. Dry cough 4. Dyspnea

Ans: Option 3 Rationales Option 1: Vasotec does not cause indigestion. Option 2: This is not a side effect of Vasotec. Option 3: Angiotensin-converting enzyme (ACE) inhibitors can cause a dry cough. Option 4: This is not a side effect of Vasotec.

The nurse is caring for a client with peripheral arterial disease. Which of the following clinical manifestations should the nurse expect the client to exhibit as a late finding? 1. Moist skin 2. Cyanotic extremity upon elevation 3. Warm extremity 4. Intermittent claudication

Ans: Option 4 Rationales Option 1: The skin is dry, not moist. Option 2: The extremity is pale when elevated and cyanotic or reddish-purple when in the dependent position. Option 3: The extremity is cool. Option 4: This is a very common symptom of late peripheral arterial disease.

A client's sinoatrial (SA) and atrioventricular (AV) nodes have failed to fire. Which of these results should the nurse expect as the ventricles take over and initiate impulses? Select all that apply. 1. Change in level of consciousness 2. Hypertension 3. Dyspnea 4. Bounding peripheral pulses 5. Tachycardia

Ans: option 1 and 3 Rationales Option 1: When the SA and AV nodes fail to fire, the ventricles initiate impulses at a rate of 20 to 40 bpm. Changes in level of consciousness is caused by inadequate cardiac output. Option 2: Hypotension, not hypertension, occurs. Option 3: Dyspnea occurs because the ventricular rate is not fast enough to meet the body's oxygen needs. Option 4: The client will have inadequate cardiac output, so bounding peripheral pulses will not be present. Option 5: Bradycardia, not tachycardia, occurs. The client's heart rate will be 20 to 40 bpm.

The nurse is caring for a client who underwent valve replacement surgery 1 hr ago. Which of the following interventions should the nurse implement? 1. Warm the client quickly. 2. Monitor vital signs every 4 hr. 3. Avoid pain medications that cause sedation. 4. Communicate using simple terms and closed-ended questions.

Ans: option 4 Rationales Option 1: The client should be warmed slowly to prevent peripheral vasodilation. Option 2: Vital signs should be monitored every 15 to 30 minutes. Option 3: The client may require pain medication, which may have a sedative effect. Option 4: The client should communicate using simple terms and closed-ended questions.

The nurse is caring for a client with thrombophlebitis. Which of the following interventions should the nurse implement? 1. Place affected extremity at or below heart level. 2. Place ice on the affected area. 3. Monitor partial thromboplastin time (PTT). 4. Administer nonsteroidal anti-inflammatory drugs (NSAIDS) as ordered.

Ans: option 4 Rationales Option 1: The extremity should be placed above heart level. Option 2: Warm, moist heat should be used. Option 3: Prothrombin time/international normalized ratio (PT/INR) is monitored; PTT monitoring is for heparin. Option 4: NSAIDS are given for discomfort.

A client recovering from cardiopulmonary bypass states, "I'm afraid something went wrong. I have tubes coming from my chest." Which response should the nurse make? 1. "The left lung is collapsed during cardiopulmonary bypass." 2. "The tubes can be used to increase your heart rate when needed." 3. "We will give medications through the tubes if your heart stops." 4. "The tubes drain remaining blood and fluid from the chest."

Ans: option 4 Rationales Option 1: The left lung is not collapsed during cardiopulmonary bypass. Option 2: Temporary pacing wires attached to the heart, not mediastinal tubes, are used to increase the heart rate. Option 3: Medications are not administered through the mediastinal tubes if the heart stops. Option 4: Mediastinal tubes are placed during cardiopulmonary bypass to drain any remaining blood and fluid from the chest.

The nurse is providing teaching to a client with strep throat about preventing rheumatic fever. Which statement made by the client indicates an understanding of the teaching? 1. "I will be given penicillin to treat the strep throat." 2. "Since I am 17, I cannot get rheumatic fever." 3. "Strep throat does not require antibiotics." 4. "I will need to get a nasal swab to see if I have strep."

Ans:1 Rationales Option 1: Using antibiotics to treat streptococcal infections can prevent rheumatic fever. Option 2: Although it is more common in ages 5 to 15, it can occur at any age Option 3: Streptococcal infections should be treated quickly with antibiotics. Option 4: A throat culture is used to diagnose strep throat.

Acetylsalicylic acid (aspirin) is prescribed for a client diagnosed with coronary artery disease before a percutaneous transluminal coronary angioplasty (PTCA). The nurse administers the medication understanding that it is prescribed for what purpose? 1. Relieve postprocedure pain. 2. Prevent thrombus formation. 3. Prevent postprocedure hyperthermia. 4. Prevent inflammation of the puncture site.

Answer: 2 Rationale: Before PTCA, the client is usually given an anticoagulant, commonly aspirin, to help reduce the risk of occlusion of the artery during the procedure because the aspirin inhibits platelet aggregation. Options 1, 3, and 4 are unrelated to the purpose of administering aspirin to this client

The nurse teaches a client at risk for coronary artery disease about lifestyle changes needed to reduce his risks. The nurse determines that the client understands these necessary lifestyle changes if the client makes which statements? Select all that apply. 1. "I will attempt to stop smoking." 2. "I will be sure to include some exercise such as walking in my daily activities." 3. "I will work at losing some weight so that my weight is at normal range for my age." 4. "I will limit my sodium intake every day and avoid eating high-sodium foods such as hot dogs." 5. "It is acceptable to eat red meat and cheese every day as I have been doing, as long as I cut down on the butter." 6. "I will schedule regular doctor appointments for physical examinations and monitoring my blood pressure."

Answer: 2, 3, 4, 6 Rationale: Coronary artery disease affects the arteries that provide blood, oxygen, and nutrients to the myocardium. Modifiable risk factors include elevated serum cholesterol levels, cigarette smoking, hypertension, impaired glucose tolerance, obesity, physical inactivity, and stress. The client is instructed to stop smoking (not cut down), and the nurse should provide the client with resources to do so. The client is also instructed to maintain a normal weight and include physical activity in the daily schedule. The client needs to limit sodium intake and foods high in cholesterol, including red meat and cheese. The client must follow up with regular primary health care provider appointments for physical examinations and monitoring blood pressure

A cardiac catheterization, using the femoral artery approach, is performed to assess the degree of coronary artery thrombosis in a client. Which priority safety actions should the nurse implement in the postprocedure period? Select all that apply. 1. Restricting visitors 2. Checking the client's groin for bleeding 3. Encouraging the client to increase fluid intake 4. Placing the client's bed in the high-Fowler's position 5. Instructing the client to move the toes when checking circulation, motion, and sensation

Answer: 2, 3, 5 Rationale: Immediately after a cardiac catheterization with the femoral artery approach, the client should not flex or hyperextend the affected leg to avoid blood vessel occlusion or hemorrhage. The groin is checked for bleeding, and if any occurs, the nurse immediately places pressure on the site and asks another staff member to contact the primary health care provider. Fluids are encouraged to assist in removing the contrast medium from the body. Asking the client to move the toes is done to assess motion, which could be impaired if a hematoma or thrombus was developing. There is no need to restrict visitors. Placing the client in the high-Fowler's position (flexion) increases the risk of occlusion or hemorrhage.

The nurse is caring for a client receiving tissue plasminogen activator (tPA). Which of the following orders should the nurse question? 1. Monitor for signs of bleeding. 2. Minimize blood draws for 24 hrs. 3. Administer acetaminophen as ordered. 4. Administer acetylsalicylic acid (aspirin) as ordered.

Answer: 4 Rationales Option 1: The client should be monitored for bleeding while on tPA. Option 2: This order is correct; blood draws should be minimized while the client receives tPA. Option 3: This order is not contraindicated with tPA. Option 4: Aspirin should be avoided because it can increase the risk of bleeding. Test Taking Tip: tPA increases the risk of bleeding.

The nurse is reviewing the assessment data of a client. Which finding is most important for the client to modify to lessen the risk for coronary artery disease (CAD)? 1. Elevated triglyceride levels 2. Elevated serum lipase levels 3. Elevated serum testosterone level 4. Elevated low-density lipoprotein (LDL) levels

Answer: 4 Rationale: LDLs are more directly associated with CAD than are other lipoproteins. LDL levels, along with levels of cholesterol, have a higher predictive association with CAD than levels of triglycerides. Lipase is a digestive enzyme that breaks down ingested fats in the gastrointestinal tract. Low rather than high levels of testosterone have a significant negative influence on CAD.

peripheral arterial disease

Disease of the peripheral arteries that interferes with adequate flow of blood.

pericardial friction rub

Friction sound heard over the fourth left intercostal space near the sternum; a classic sign of pericarditis.

homograft

Graft from a donor of the same species as the recipient. hopelessness: Subjective state in which a person sees limited or unavailable alternatives; lacking energy.

acute coronary syndrome

Group of conditions, including unstable angina, non-Q-wave myocardial infarction, and ST segment elevation myocardial infarction, caused by a lack of oxygen to the heart muscle.

coronary artery disease

Narrowing of the coronary arteries sufficient to prevent adequate blood supply to the myocardium.

The nurse is caring for a client undergoing an angiography. After injection of the dye, the client states, "I have a hot, burning feeling." Which action by the nurse is the priority? 1. Deliver supplemental oxygen 2. Administer diphenhydramine 50 mg intravenous (IV) 3. Prepare for endotracheal intubation 4. Monitor the client

Option 4 Rationales Option 1: There is no indication to administer supplemental oxygen at this time. Option 2: There is no indication to administer diphenhydramine 50 mg IV at this time. Option 3: There is no indication to prepare for endotracheal intubation at this time. Option 4: The client is having a normal response to the dye. The nurse should continue monitoring the client at this time.

The nurse is preparing a client for a cardiac catheterization. Which of the following is the most important piece of data to know prior to the procedure? 1. Current height and weight 2. Allergies to iodine, shellfish, or contrast dye 3. Baseline pulse assessment 4. The client's last oral intake

Rationales Option 1: Although current height and weight are essential to know, allergies are most important Option 2: Allergies are the most important piece of data to know; giving a client dye when the client is allergic can cause severe adverse reactions or death. Option 3: Although it is important to have a baseline assessment of pulses, allergies are most important. Option 4: Although this is important to know (NPO for 8 hours prior to the test), allergies are a higher priority. Test Taking Tip: All of these pieces of data are important, but one is MOST important.

The nurse is caring for a group of clients. Which client should the nurse see first? 1. A client with a blood pressure of 200/104 in kidney failure 2. A client with a blood pressure of 180/80 with a potassium of 3.0 mEq/L 3. A client with a blood pressure of 190/100 who reports intermittent claudication 4. A client with a blood pressure of 180/90 who reports chest pain

Rationales Option 1: This blood pressure is very elevated, but since the client is in kidney failure (a complication of high blood pressure), the client's condition is likely chronic. Option 2: This blood pressure is elevated and the potassium is slightly low, but this is not the first client to see. Option 3: This blood pressure is elevated, but the client reports pain in the legs with activity; this client is not the priority. Option 4: This client has elevated blood pressure (not as elevated as some) but is reporting chest pain, which could be caused by a myocardial infarction (MI). This client is unstable and should be seen first. Test Taking Tip: Clients should be seen in order from most unstable to least unstable.

The nurse is teaching a client with heart failure about ways to prevent orthopnea. Which statement made by the client indicates an understanding of the teaching? 1. "I should sleep with two or three pillows so I can breathe easier." 2. "I should lie in a flat position as long as I am able to." 3. "I can lie flat as long as my legs are elevated." 4. "I should only use one pillow to sleep at night."

Rationales Option 1: Two or more pillows should be used for sleeping when orthopnea is present. Option 2: Dyspnea occurs in clients with orthopnea when lying flat. Option 3: A client with orthopnea has dyspnea that worsens when lying flat. Option 4: A client with orthopnea should sleep with two or more pillows. Test Taking Tip: The supine position is not recommended for clients with orthopnea.

myocarditis

The inflammatory process that causes nodules to form in the myocardial tissue; the nodules become scar tissue over time. Inflammation of the heart muscle.

cardiac tamponade

The life-threatening compression of the heart by the fluid accumulating in the pericardial sac surrounding the heart.


Ensembles d'études connexes

aspirin (acetylsalicylic acid, ASA)

View Set

ATI Pharmacology Across the Lifespan

View Set

Social Change Unit 2 (no vocabulary)

View Set

Ch.9.2 Chemical Reactions 8th grade

View Set